You are on page 1of 107

Casebook

Trama
Club de Consultora

2015
Trama es una organizacin de estudiantes que nace en 2012 con
dos objetivos principales, por un lado acercar el mundo laboral a la
facultad y por otro generar en los jvenes inters en las temticas
de actualidad.
Desde sus inicios la organizacin cuenta con tres clubes, uno de
finanzas, uno de consultora y uno de emprendedores. Cada club
tiene un ciclo de actividades cuatrimestral, abierto a todos los es-
tudiantes, que brinda la posibilidad de conocer de cerca cada uno
de estos campos.
En el club de consultora se desarrollan encuentros para conocer
las herramientas que utiliza un consultor en su actividad laboral, y
las charlas estn a cargo de experimentados profesionales.
Adems, se realiza en conjunto con Bain&Company una competen-
cia de casos durante el mes de mayo, en la que participan alumnos
de diversas universidades y carreras.
El club de finanzas tambin tiene su agenda cuatrimestral, que va
tocando diferentes temas desde mercados burstiles hasta finan-
zas personales. Tambin se lleva a cabo una simulacin de inver-
sin en bolsa junto a clubes de finanzas de las universidades de
San Andrs y Di Tella.
El club de emprendedores se caracteriza por desarrollar encuentros
tanto tericos como prcticos, de la mano de mltiples emprende-
dores exitosos que estn dispuestos a transmitir su experiencia a
los estudiantes interesados en este campo.
El principal evento que organiza el club de emprendedores es el
BootCamp Trama, un evento emprendedor de tres das de traba-
jo intensivo, al que asisten jvenes estudiantes de todo el pas y
pueden trabajar sus ideas as como tambin conocer herramientas
tiles a la hora de emprender.

Trama es consciente que las redes son un fenmeno social y tecno-


lgico que han aparecido en los ltimos aos para cambiar nuestra
forma de pensar y comportarnos. Empresas, profesores y alumnos
que conformamos Trama compartimos la filosofa de que el cono-
cimiento de uno es el conocimiento que tiene su red de contactos y
esa es la principal razn de existir de esta organizacin.
Agradecimientos

Queremos agradecer a todos los que hicieron posible la realizacin de


este libro.
En primer lugar queremos agradecerle al ITBA, sin su apoyo Club Trama
no podra realizar todas las actividades para el beneficio de sus alumnos.
Tambin queremos agradecerle a nuestro sponsor Bain& Co. quien des-
de el comienzo del club deposit su confianza en nosotros y constante-
mente nos da asesoramiento para nuestras actividades.
De igual manera queremos agradecerle a todas las empresas que forma-
ron parte de este casebook tanto aportando casos, facilitando encuestas
para su desarrollo y aconsejando en diversas partes del mismo.
Tambin nos gustara reconocer el esfuerzo y dedicacin especial de To-
ms Berges, Martn Carrica, Matas Micheloni y Pablo Peralta quienes ju-
garon un papel clave en la realizacin del libro.
Agradecemos el apoyo de la comunidad de alumnos y egresados del
ITBA quienes aportaron ideas y ayudaron con nuestras encuestas duran-
te el proceso.
Finalmente nos gustara agradecer al resto del equipo de Trama, espe-
cialmente a nuestros directores Mariano Carrica y Ramiro Kljenak, por su
apoyo y voluntad durante el transcurso del proyecto.

Muchas gracias a todos,

Club Trama de Consultora.

Coordinadores 2014 - 2015

Tomas Waisman Bruno Cortinovis


Trama
Conectando conocimiento
Sponsors
Contribuidores de Casos
ndice
- Club de Consultora 2
- Agradecimientos 3
- Sponsors 7
- Contribuidores de Casos 9
1. Introduccin: qu informacin tengo que conocer? 13
1.1. Cmo usar este libro? 13
1.2. Cmo son los procesos tpicos derecruiting? 14
Evento de recruiting
Recepcin de CVs
Examen
Rondas de entrevistas
1.3. Cmo prepararse? 17
Metodologa
1.4. Cules son las actividades importantes del ao? 18
Cronograma de eventos
Observaciones respecto al cronograma
1.5 De dnde puedo estudiar? 20
Biblioteca
2. Estudio sobre los alumnos del ITBA 23
2.1. Qu buscan las consultoras? 23
Qu habilidades espera encontrar una
consultora en sus aplicantes?
2.2. Qu errores son comunes en el proceso? 27
Armado del CV
Examen tipo GMAT
Entrevista de Casos
Evaluacin de Fit con la empresa
2.3. Qu aconsejan los egresados del ITBA? 32
3. Frameworks 35
3.1. Anlisis de la rentabilidad 36
3.2. Situacin de negocios 38
3.3. M&A (Mergers and Acquisitions) 45
3.4. Estrategias de Marketing 47
3.5. Mini Frameworks 49
4. Casos de Prctica 51
Caso Nmero 1: GREEN BEANS NASCAR TEAM 51
Caso Nmero 2: Banco Alfa 58
Caso Nmero 3: Starbucks Coffee 64
Caso Nmero 4: Autopartista 74
Caso Nmero 5: Licitacin de semforos 81
Caso Nmero 6: SOUTH AMERICAN AIRLINES (SAA) 87
Caso Nmero 7: BeerBusch 92
Caso Nmero 8: Steel 96
Caso Nmero 9: Salud 102
13

1. Introduccin: qu informacin
tengo que conocer?
1.1. Cmo usar este libro?

El objetivo principal de este libro es servir de gua para los


alumnos del ITBA interesados en aplicar a empresas de con-
sultora estratgica. Como tal, la informacin que se provee
es necesaria, pero no suficiente.

Se adopt un ndice con una estructura y formato intuitivos


para poder responder preguntas que generalmente surgen al
iniciar el proceso. En primer lugar, se brinda una descripcin
general del mismo, luego se analiza en detalle lo que se espera
del aplicante y, finalmente, se brindan herramientas para la re-
solucin de casos, junto con ejemplos prcticos tiles.

Es preciso recordar que este libro es nicamente una he-


rramienta, y la sola lectura del mismo no asegura bajo nin-
El libro es una
guna circunstancia la obtencin de una oferta de trabajo. herramienta y no
Como sugerencia principal, para lograr un mejor resultado, asegura una buena
performance en las
se aconseja practicar las entrevistas con una o varias per- entrevistas.
sonas. Asimismo, es recomendable practicar con personas
que cuenten con experiencia en las mismas, ya que los es-
tilos para entrevistar varan y, de este modo, ser posible
recibir diferentes comentarios. El objetivo de este libro es,
en consecuencia, poder recrear una situacin de entrevista
laboral lo ms fielmente posible, y as poder preparar com-
petentemente al aplicante para tal.

Casebook Trama 2015


14

1.2. Cmo son los procesos tpicos de recruiting?

Los procesos de seleccin de la mayora de las consultoras


son muy similares en su estructura. Los mismos estn for-
mados por tres instancias principales: Recepcin de CVs,
Examen y Rondas de entrevistas.

Cada una de estas etapas busca conocer distintas caracte-


rsticas de los aplicantes.

RECEPCIN ENTREVISTA
EXAMEN
DE CVS DE CASOS

Formacin acadmica Capacidad de Capacidad analtica y


Logros razonamiento analtico estructuracin para
(test tipo GMAT) resolver problemas
Actividades
Comprensin de ingls Energa y empuje
extracurriculares
Buen sentido comn
Experiencia profesional
Fit e inters en consultora
Ingls

Asociado al comienzo de los procesos algunas consultoras


optan por realizar eventos de recruiting.

Evento de recruiting

En algunas ocasiones, las consultoras optan por hacer even-


tos en hoteles, o en sus propias oficinas, con el objetivo de
conocer a los candidatos sin la presin de las entrevistas.

Estos eventos consisten en una presentacin de la empre-


sa, seguida de detalles sobre el proceso de recruiting y de
Los eventos son un momento informal de charla con los representantes de la
una buena manera empresa que se encuentren all.
de conocer a las
consultoras.
Dichos eventos no son obligatorios para el proceso, pero es
15

sabido que ponen en evidencia una muestra de inters y


compromiso por el mismo. Se recomienda asistir a los even-
tos de las consultoras que resulten de inters para el apli-
cante, con el fin de despejar todas sus dudas, ya que ese es
el objetivo de esta etapa.

Recepcin de CVs

La forma de comenzar con el proceso es con el envo del


CV. La misma es una etapa muy importante que suele ser
muy descuidada por los alumnos.

ste es la primera presentacin formal ante la empresa, y puede


Un Buen CV es
significar continuar en el proceso o quedar afuera directamente. el primer paso
para un proceso
Existen consultoras que, junto a ste, solicitan una carta de exitoso.
presentacin donde el aplicante cuenta las intenciones que
tiene para trabajar en esa empresa en particular. Este do-
cumento es igual de importante que el CV, as que tambin
requiere dedicarle tiempo y trabajo.

Examen

El formato del examen que se toma vara entre consulto-


ras. Sin embargo, todas tienen como referencia al examen
GMAT tomado en la etapa de ingreso de las principales
business schools del mundo.

El GMAT es un examen que mide capacidades verbales,


matemticas y analticas que los alumnos han desarrollado
durante sus estudios. Es importante destacar que el examen
no mide conocimiento de negocios, habilidades de trabajos
anteriores, conocimientos especficos de alguna carrera, ni
habilidades de creatividad o interpersonales.

Casebook Trama 2015


16

Rondas de entrevistas

Los procesos tpicos de entrevistas en las consultoras ms


grandes siguen la siguiente estructura.

Entrevistador Consultor senior Partner


Tpico : Manager

Foco de Habilidades Habilidades soft


Evaluacin : cuantitativas / analticas Fit con consultora
Estructura de
razonamiento

Entrevistas : Tpicamente 2 entrevistas 2/3 entrevistas ( y posibili-


dad de 3era ronda )

Historia Personal 5 minutos


Es importante
Durante esta parte, el entrevistador querr saber ms acerca
contar una historia de los intereses del candidato y de cmo estos estn alineados
que resalte tus
habilidades.
con los de la consultora. El entrevistador dar la oportunidad al
aplicante de darse a conocer y estar curioso por escuchar su
motivacin para entrar a la compaa.

Caso de Estudio 25 minutos


Durante las entrevistas de casos, los entrevistadores obser-
van cmo los candidatos analizan, estructuran y resuelven
problemas de negocios. Las entrevistas de casos son un
excelente indicador de cun bueno podra ser un futuro con-
sultor en cuestiones de problem solving y personalidad.

Las principales habilidades de son:


Analizar problemas de una forma estructurada
Reconocer puntos crticos
17

Colectar, seleccionar y organizar la informacin


Formular una hiptesis y probarla
Pensar creativamente
Tener business sense y sentido comn
Un caso puede separarse en las siguientes etapas:

IDENTIFICAR ESTRUCTURAR ANALIZAR RECOMENDAR


PREGUNTAS CLAVE EL PROBLEMA EL CASO SOLUCIN

Usar un framework Proactivamente llegar a


simple y relevante para una respuesta
guiar la discusin Consolidar los distintos
Descomponer el elementos de anlisis en
problema complejo en una respuesta coherente
las partes que lo Recomendar un plan de
componen accin prctico para
Desarrollar una hiptesis obtener resultados
Compartir premisas en
voz alta

Preguntas y Respuestas 5 minutos


En este momento de la entrevista, el aplicante tendr la Se espera pregun-
tas inteligentes,
oportunidad de hacerle preguntas al entrevistador. Es una nada que se pueda
excelente oportunidad para poder conocer los recursos hu- responder buscan-
manos y la cultura de cada compaa. do en internet.

1.3. Cmo prepararse?


Metodologa

A continuacin se describe una serie de pasos para prepararse


para una entrevista. Es importante destacar que la estrategia

Casebook Trama 2015


18

10% a la prepara- que se presenta busca ser lo ms completa posible, para que
cin del GMAT el aplicante interesado tenga una visin completa de los recur-
70% a la prepara-
cin de Casos
sos disponibles.
20% a la prepara-
cin de Fit y CV. Se recomienda que cada aplicante arme su propio plan, de
modo que se adece a sus propias conveniencias. A modo
de gua, se hizo una encuesta entre egresados del ITBA que
aplicaron a procesos para saber cunto tiempo dedicaron a la
preparacin y cmo fue su performance. A partir de los resul-
tados, se recomienda dedicarle al proceso una preparacin de
entre 2 y 3 semanas. Tambin es importante resaltar el porcen-
taje de dedicacin a cada parte del proceso.

PRIMER DESARROLLO DE
ACERCAMIENTO HABILIDADES

Eventos Trama: Eventos Trama: Eventos Trama:


- Introduccin a la - Mock exam - Prctica de casos
consultora interclubes de
Casos: consultora
- Crack the case
- Entrevistas interactivas
Casos: en pginas de Casos:
- Videos en Youtube. consultoras - Entrevistas online con
( Uso de Frameworks y - Entrevistas entre personas de otros
Mock Interviews ) compaeros pases
- Framework de - Entrevistas con
Casebooks Gmat: egresados consultores
- Libros con material y ( el club tiene una lista
Gmat: exmenes de egresados
- Pginas web con voluntarios )
exmenes online

1.4. Cules son las actividades importantes del ao?


Cronograma de eventos

A continuacin se presenta un cronograma con los eventos


ms relevantes relacionados a la consultora estratgica.
19

La idea de ste es orientar al alumno del ITBA sobre las


fechas del ao en las cuales se realizan los procesos de las
consultoras, y que as dicho alumno pueda organizarse de
manera ms eficiente.

Observaciones al respecto del cronograma:

Procesos de recruiting:
Primero, es vlido destacar que existen consultoras que
realizan procesos durante todo el ao y otras en fechas es-
pecficas. Las resaltadas en el cronograma son las fechas
puntuales de estas ltimas, que coinciden con la poca de
mayor difusin por parte de las otras consultoras.

Es importante distinguir los distintos procesos durante el


ao. Los procesos ms representativos son los de agos-
to-septiembre. Estos son realizados de forma masiva con
una mayor convocatoria que los de abril-mayo.

Rondas de entrevistas:
Las rondas de entrevistas de las empresas pueden variar,
pero generalmente son entre una y dos semanas despus
de que comenzaron los procesos, por lo tanto hay poco
margen para dedicarle a la prctica de casos luego del co-
mienzo de la aplicacin.

Eventos de Trama:
Entre los eventos de Trama se encuentran dos totalmente

Casebook Trama 2015


20

enfocados en ayudar a los alumnos en el proceso masivo


de agosto-septiembre. El primero es una charla de armado
de CV e iniciacin del proceso, mientras que el segundo es
una sesin de prctica de casos con ayuda de consultores.
Sumado a esto, se organizan reuniones de prcticas con
otros clubes de consultora de Buenos Aires.

Adems, durante el resto del ao, se realizan otros eventos


para introducir a los estudiantes a la consultora y permitirles
conocer herramientas que los consultores utilizan cotidianamente.

1.5. De dnde puedo estudiar?


Biblioteca

GMAT:
Pginas de GMAT:
o www.majortest.com/gmat/problem_solving.php
o http://www.platinumgmat.com/
o http://freegmattest.net/

Ejemplos de las pginas de las consultoras (Bain, BCG,


Mckinsey)

Libros de GMAT:
o Master the GMAT by Petersons
RECOMENDADO POR EL CLUB
o Cracking the GMAT by Princeton Review
o McGraw-Hills GMAT by James Hasik, Stacey Rudnick,
Ryan Hackney
Record que el
club de consul- o Kaplan GMAT Premier by Kaplan RECOMENDADO POR EL CLUB
tora cuenta con
recursos tiles para o The Official Guide for GMAT Review by Graduate
practicar, de acce- Management Admission Council
so libre y gratuito.
o Manhattan GMAT Preparation Guide by Manhattan GMAT
21

Entrevista de Casos:
http://mconsultingprep.com

Videos de Victor Cheng

Casebooks de otras universidades (MIT, Harvard, Wharton,


etc.)

http://masterthecase.com/case-interview-casebooks/
RECOMENDADO POR EL CLUB

Cornell University: http://cornelluniv.cqinteractive.


com/cqiaccess/member_dashboard.cfm RECOMENDADO POR EL CLUB

Casos interactivos (pginas de Bain, BCG, Mckinsey)

Libros:
o Case in Point by Marc P. Cosentino
o Case Interview Secrets by Victor Cheng
o The Vault Guide by vault.com

App para celular de AUB Consulting

Lo ms importante
es practicar con
amigos y
consultores.

Casebook Trama 2015


22
23

2. Estudio sobre los alumnos


del ITBA
2.1. Qu buscan las consultoras?

El objetivo de esta seccin es que el aplicante conozca qu


habilidades buscan las consultoras y cul es la visin que
tienen sobre los alumnos del ITBA.

Qu habilidades espera encontrar una consultora en


sus aplicantes?

Las consultoras buscan identificar y evaluar a los entrevistados


en funcin de cuatro pilares fundamentales

Capacidad Analtica

Los alumnos del ITBA suelen tener desempeos muy bue-


nos a la hora de demostrar sus capacidades analticas. Los
entrevistadores coinciden en que el perfil tpico de los alumnos

Casebook Trama 2015


24

tiene una tendencia a ser muy estructurado y comprometido


a la hora de resolver un desafo. Sumado a esta caracters-
tica, la formacin de los estudiantes hace que la mayora
se sienta cmodo y tenga los recursos necesarios para re-
solver clculos de relativa complejidad. Es entonces este
aspecto una fortaleza para los alumnos del ITBA.
Se considera como anlisis el hecho de explorar y enten-
der el funcionamiento de algo a partir de sus componen-
tes. Para explotar al mximo esta habilidad, es fundamen-
tal tener conocimiento y entrenamiento de un pensamiento
MECE (Mutuamente Excluyente y Colectivamente Exhaus-
tivo). MECE es un principio fundamental para estructurar y
seccionar cualquier problema o idea. Este concepto es muy
importante en la consultora estratgica, y es por eso que se
vuelve sumamente importante en una entrevista de casos.

Se considera que un seccionamiento es MECE cuando:

Las partes en las que se secciona un TODO no se


superponen.
La suma de todas las partes es igual al TODO.
Para ms informacin: http://mconsultingprep.com/case-in-
terview-mece/

El segundo punto importante para un buen anlisis es for-


mular correctamente la HIPTESIS. El tiempo es un bien es-
caso en una entrevista y, en la medida en que el entrevistado
sea capaz de validar o rechazar las hiptesis planteadas,
Egresados que pa- estar aprovechando mejor su tiempo.
saron por procesos
reconocen que al-
Hay cuatro reglas bsicas para formular correctamente
gunos de los errores
que cometieron en una hiptesis:
este aspecto fueron:

No utilizar frameworks Las hiptesis deben respetar el framework elegido.


de la forma correcta. Las hiptesis deben ir TOP to DOWN.
Errores de clculo.
Toma de decisiones
Siempre tener una hiptesis. Tener una hiptesis
apresuradas. errnea es mejor que no tener nada.
Falta de apertura
Las hiptesis deben basarse en informacin que
a ms soluciones.
tenemos.
25

Liderazgo y Trabajo en Equipo

Liderazgo y trabajo en equipo son habilidades inherentes a


cada aplicante. Esta caracterstica es demandada por cual-
quier empresa en la actualidad, ya que se busca en los j-
venes egresados potencial para tener un rol de liderazgo a
futuro. Para poder destacarse en este aspecto, es necesario
detectar experiencias pasadas donde se hayan demostrado
estas habilidades para llevar adelante una iniciativa en bus-
ca de resultados.

Muchos egresados que participaron de entrevistas recibie-


ron, como feedback, que les falt seguridad y personalidad
a la hora de empujar la recomendacin final. Es importante SEGURIDAD y
que el entrevistado entienda que el entrevistador est espe- PERSONALIDAD
rando recibir una recomendacin, y no saber si el aspirante es lo que les suele
faltar a los alumnos.
usa bien o mal un framework.

Los pilares Business Sense y Comunicacin Interper-


sonal merecen especial atencin ya que, para la mayor
parte de los entrevistadores, estos suelen ser un punto
dbil de los estudiantes del ITBA.

Business Sense

Con respecto al Business Sense, las consultoras no buscan


que el aplicante sea un experto en cada industria. No obs-
tante, lo que se desea es que el entrevistado sea capaz de
detectar drivers que muevan al negocio. En otras palabras,
lo que se pretende es que se tenga una capacidad de Smell
Money: ser capaz de detectar cmo el negocio genera in-
gresos, dnde tiene los mayores costos, y la naturaleza de
sus productos, entre otras cosas. SMELL MONEY
Cmo genera
dinero la empresa?
A modo de ayuda, se recomienda ver el casebook de Wharton Dnde gasta
innecesariamente?
2010 en la seccin Industry Snapshots (a partir de la pag. 39). Cules son los
All se muestra a continuacin un paneo general de distintas in- drivers que mue-
ven al negocio?
dustrias. Es importante que el alumno que mire este cuadro no

Casebook Trama 2015


26

pierda tiempo en aprender en detalle cada caracterstica.


La idea del cuadro es servir de gua para que el alumno se
familiarice con distintos negocios.

Nuevamente, se hace hincapi en el hecho de que el entre-


vistado debe tener como prioridad dar una recomendacin
Saber usar un fundamentada para el caso de negocios que le presentaron.
framework no es El alumno debe explotar al mximo los insights obtenidos
suficiente si no se
complementa
de su anlisis para lograr formar una buena recomendacin.
con una visin Saber usar un framework no es suficiente si no se comple-
estratgica. menta con una visin estratgica.

Comunicacin Interpersonal

En cuanto a la Comunicacin Interpersonal, es importan-


te que el entrevistado sea capaz de tener claridad y orden
a la hora de comunicar un mensaje. Las capacidades para
presentar y entrevistar son habilidades sumamente necesa-
rias para el consultor; por esta razn, es uno de los puntos
que se evala. Es recomendable que el aplicante muestre
seguridad tanto en la comunicacin verbal como en la no
verbal. Para este pilar, se recomienda apalancarse en los
trabajos que se exponen durante la carrera universitaria; es-
tos se encuentran en un ambiente seguro donde se pueden
experimentar distintas estrategias de presentacin de ideas.
PENSAR EN VOZ
Un error repetido en los estudiantes del ITBA es no pen-
ALTA demuestra la sar en voz alta. Es importante que el entrevistador entienda
forma de razonar cules son las ideas del entrevistado y cmo las estructura
de cada uno.
para poder encontrar una solucin.

Extras

Por ltimo, se nombraron en menor medida otras capacida-


des como:

Determinacin y compromiso. Mostrar inters


genuino en consultora. Tener objetivos claros y
determinacin para lograrlos.
27

Madurez, seguridad y Pasin.


Time management. Saber priorizar actividades y
organizar tiempos.
Curiosidad y Creatividad. Ser capaz de tener una
apertura para cuestionar y salir del status quo.

2.2. Qu errores son comunes en el proceso?

Armado del CV

Es importante que el aspirante tenga en claro que el objetivo


de un CV debe ser contar una historia de impacto, enfocada
y atractiva para la compaa a la que estamos aplicando, y
NO una mera lista de actividades del pasado.
Todos los CVs tardan un tiempo en lograr madurar y conver-
tirse en competitivos, por lo que es recomendable dedicarle
tiempo al mismo e ir mejorndolo en varias iteraciones. Ob-
tener feedback de profesores o familiares suele ser la mejor
manera de lograr un CV capaz de contar una buena historia
sobre uno mismo.

Casebook Trama 2015


28

Seguidamente se enumera una serie de tips para el armado


de un CV:

CV en una sola pgina.


Considerar el formato general del CV (fuente,
distribucin, alineacin del texto).
La foto no es necesaria, pero si se incluye debe ser
una foto formal.
No hacer un collage de colores, evitar muchas
fuentes y tamaos de letras.
Informacin de contacto actualizada y pertinente.
Formato de archivo (.pdf)
Evitar dejar espacios vacos.
Comenzar con lo ms relevante: para aquellos con
experiencia laboral resulta ms conveniente poner
primero su trabajo ms reciente, mientras que para
aquellos sin experiencia conviene poner primero la
formacin acadmica.
La informacin adicional, hobbies, etc. suelen servir
si demuestran cualidades como liderazgo, espritu
de equipo, capacidades tcnicas, etc.
Ser inteligente en la construccin de las secciones
del CV: la seccin de informacin adicional/hobbies
no debe ser ms extensa que la experiencia laboral
o la formacin acadmica.

Examen tipo GMAT

Algunas categoras tpicas del GMAT usadas para evaluar a


los aplicantes son:
29

Algunos tips para esta etapa son:

Leer cuidadosamente las preguntas. Asegurarse de


que en realidad se est respondiendo a la pregunta
formulada. A veces, la prueba puede tener
preguntas confusas, o algo que pasa desapercibido.
Usar un borrador para todas las preguntas. No
importa que la pregunta sea fcil, escriba sus
clculos y razonamiento. Esto ayuda a evitar
errores fciles.
No atraparse en clculos complejos. Si percibe que Cuando uno queda
atrapado en un
algn clculo es excesivamente complejo, puede calculo comple-
significar que tom un abordaje ms difcil que el jo puede ser til
necesario para resolver el problema. pasar a la siguiente
pregunta y volver a
Estimar un valor prximo. En muchos problemas, intentar resolverla
las alternativas sern numricamente muy distantes. ms tarde.

Casebook Trama 2015


30

En estos casos con un valor aproximado de las


respuestas se puede elegir un resultado y
aprovechar el tiempo para otra pregunta
Usar un proceso de eliminacin. Incluso si no
pods responder la pregunta con precisin, es
posible tener xito en la eliminacin de alternativas
porque sus valores son absurdos
Calcular el tiempo que se tiene por pregunta y no
disponer ms tiempo que el necesario en cada una.
De ser posible, practicar previamente con los
modelos que ofrecen las empresas. Suelen ser muy
100 % de los representativos del examen real.
alumnos entrevis-
tados coincidieron
en que la mejor Entrevista de Casos
forma de mejorar
es practicando.
Los errores ms comunes en esta etapa son:
31

Evaluacin de Fit con la empresa

Se entiende por Fit con la empresa a cmo encaja la


persona entrevistada con los valores de la empresa y el
equipo actual.

Un ejemplo para explicar esto es el Test del aeropuerto. B-


sicamente el entrevistador piensa si podra pasar cinco horas
con el entrevistado mientras esperan para tomar un vuelo.

La parte Fit de una entrevista suele ser una seccin descui-


dada por los alumnos del ITBA.

Si bien no existe una etapa aislada para evaluar el Fit, exis-


ten cuatro momentos donde este factor toma protagonismo:

Presentacin personal
Preguntas que se realizan al entrevistador
Conocimiento de la consultora donde se aplica
Comportamiento y etiqueta

A continuacin se presentan errores comunes que los entre-


vistados suelen cometer en cada rea

Casebook Trama 2015


32

Las dos primeras reas son las ms importantes para la ma-


yora de los entrevistadores encuestados.

Presentacin personal

Se recomienda practicar las preguntas frecuentes para la par-


te de presentacin personal. Un buen lugar para encontrarlas
es el apndice E del casebook de Kellogg del ao 2004.

Es necesario tener una idea clara en los planes personales para


el futuro y por qu consultora sera el prximo paso lgico.

Otros factores mencionados por egresados que pasaron por


los procesos son aprovechar experiencias, como intercam-
bios en el extranjero, pasantas, idiomas y deportes, siempre
y cuando demuestren cualidades que buscan los consultores.

Preguntas que se realizan al entrevistador

A los entrevistadores les gustan las preguntas que demuestran


entusiasmo o estn relacionadas con sus carreras profesionales.

Siempre trabajaste en consultora? Por qu


cambiaste/no cambiaste?
Cmo es trabajar en la consultora en particular?
Y en esa oficina? (si es que hay varias).
Cmo son los programas de training de la
consultora? (si es que los tiene)
Qu le pareci su MBA? (si es que lo tiene)

2.3. Qu aconsejan los egresados del ITBA?

Hay tres puntos que se buscan resaltar en la retrospectiva


de los que ya aplicaron

La respuesta ms significativa fue que se podra haber


practicado ms. Si bien llega un punto en la curva de apren-
33

dizaje donde ya no se ve reflejado el esfuerzo en la mejora,


tambin se dijo que esto podra haber dado ms confianza
a la hora de la entrevista. Muchos aplicantes destacan que
haber estado ms calmados les podra haber jugado a favor
en la entrevista.
En segundo lugar, no perder el foco al encuadrar el proble-
ma. Es importante seguir una metodologa clara y correcta
para poder desempearse mejor ante los evaluadores.
Preparase mejor para el GMAT. Muchos advierten haberse
confiado en esta seccin y quedar fuera del proceso por
haber hecho foco directamente en la parte de entrevistas. Si
bien esta parte no requiere tanta prctica como las entrevis-
tas de casos, no deja de ser una etapa a superar.

Casebook Trama 2015


34
35

3. Frameworks
Qu es un framework? Dicho anglicismo se refiere a un mar-
co de trabajo que se puede tomar en cuenta para resolver los
casos evaluados en las entrevistas, es decir, un conjunto es-
tandarizado de conceptos, prcticas y criterios para enfocar
un tipo de problemtica particular que sirve como referencia
para enfrentar y resolver nuevos problemas de ndole similar.
En resumen, un framework da informacin necesaria para
encarar la resolucin de casos de estudio, de manera ms
eficiente y dinmica.
Sin embargo, se debe tener en cuenta el hecho de que no hay
una respuesta nica, absolutamente certera, para cada caso
de estudio provisto, razn por la cual el aplicante debera ser
capaz de ir alternando fluidamente de un framework a otro,
evaluando cul es el que mejor se ajusta al anlisis, para as
lidiar gilmente con la resolucin del caso y con las sugeren-
cias que te hace el entrevistador. En las siguientes pginas se
presentan los mencionados frameworks, explicando en qu
situaciones de negocio se recomienda emplear cada uno y,
posteriormente, describiendo los en profundidad.
Para empezar, hay cinco tems que se pueden pedir que evale
el aplicante durante el caso.

1) Rentabilidad del negocio


2) Situaciones de negocio
3) M&A (Mergers & Acquisitions)
4) Estrategias de marketing
5) MiniFrameworks

Casebook Trama 2015


36

3.1. Anlisis de la rentabilidad

Este anlisis debera ser empleado en preguntas acerca de la


obtencin de beneficios, acerca de las operaciones, o acerca
de nuevas oportunidades de negocio para la empresa.

VOLUMEN
INGRESOS
PRECIO
RENTABILIDAD
FIJOS
COSTOS
VARIABLES

Preguntas generales sobre este tema

Cules son las fuentes de ingreso de la empresa?


Hay ms de un flujo de beneficios (profit streams)?
Hay ms de un flujo de ingresos (revenue streams)?

Si existe ms de un flujo, analizar cada uno por separado. La


clave es centrarse en el mix de ventas, y en los incentivos a
las fuerzas de venta.

La siguiente frmula, que se deduce del framework visto en


el tem 1, es esencial para el correspondiente anlisis:

Beneficio = Ingresos Totales Costos Totales

donde Ingresos Totales = Precio*Cantidad


y donde Costos Totales = Costos Fijos + Costos Variables
37

En base a dicha expresin, podemos deducir dos princi-


pios generales:

Incrementar el ingreso total incrementando el pre-


cio y/o la cantidad.
Entender la estructura de costos y tratar de reducir
los que sean ms significativos.

La pregunta principal para analizar los ingresos de una em-


presa es hasta cundo incrementar el precio, y hasta cun-
do la cantidad?

Las siguientes preguntas sirven para entender la estructura


de costos de una empresa:

Casebook Trama 2015


38

- Cules son los principales drivers de costos dentro


de la empresa?
- Qu costos son fijos y cules son variables?
- Cmo han cambiado los costos a travs del
tiempo?
- Cul es la estructura de costos de las empresas
competidoras?

3.2. Situacin de negocios

Los siguientes frameworks sirven para analizar las entradas


a nuevos mercados, el lanzamiento de nuevos productos,
39

los nuevos negocios, cmo crecer, cules son las estrate-


gias de negocios, qu es el turnaround, y cmo mejorar la
valoracin de la compaa.

3C & P

Casebook Trama 2015


40

Anlisis de la industria:

Las cinco fuerzas de Porter, junto a una sexta fuerza que


generalmente se agrega al anlisis, la cual es el gobierno.
41

Para el anlisis de Porter, analizar la siguiente informacin en


cuanto a las barreras de entrada, los productos sustitutos, los
proveedores, los consumidores y las empresas rivales.

Casebook Trama 2015


42
43

Anlisis de la Cadena de Valor

Usar la cadena de valor (Supply Chain) para:

1. Identificar componentes de costos.


2. Identificar componentes de ingresos.
3. Relacionar actividades de la empresa con sus
competencias.

Casebook Trama 2015


44

La siguiente grfica corresponde a la cadena de valor de Porter.

Actividades primarias

Logstica interna: recepcin, almacenamiento y depsito de


insumos, control de inventarios, manejo de materiales, pla-
nificacin de los vehculos y retorno a los proveedores.

Operaciones: transformacin del input en producto final (ma-


quinado, empaquetamiento, ensamblaje, mantenimiento,
evaluaciones de producto, estampado y otras instalaciones
operativas).

Logstica externa: distribucin del producto terminado (de-


psito de productos terminados, manejo y despacho del
producto, procesamiento de rdenes y planificacin de la
entrega del producto).

Marketing y ventas: estimula y facilita el hecho de que los con-


sumidores compren el producto (publicidad, fuerza de ven-
tas, cotizacin, eleccin del canal de ventas, relaciones del
canal y fijacin de precios).
45

Servicios: se encargan de mantener o mejorar el valor del


producto, luego de la venta (instalacin, reparacin, entre-
namiento, suministro de partes, ajuste final de producto).

Actividades de apoyo

Aprovisionamiento: compra de material prima, de suministros


y de otros consumibles, as como la adquisicin de activos.

Desarrollo de la Tecnologa: know-how, procedimientos, input


tecnolgico necesario en cada actividad de la cadena.

Gestin de Recursos Humanos: seleccin, promocin y evalua-


cin de personal; beneficios, desarrollo de la gestin y de
las relaciones laborales.

Infraestructura de la Empresa: administracin general, planea-


miento, Finanzas, Contabilidad, Legales, relaciones guber-
namentales y control de la calidad.

3.3. M&A (Mergers and Acquisitions)

Este framework es especficamente til cuando la empresa


tiene que decidir si entrar a un nuevo mercado por adqui-
sicin, es decir, comprando un paquete accionario de otra
sociedad para apropiarse de la misma, sin necesariamente
fusionar sus patrimonios.

Casebook Trama 2015


46

En principio, se debe analizar cmo se encuentra la empre-


sa por s sola, independientemente de la compra. Es til ver
la posicin relativa de las mismas.

A continuacin, considerar si el negocio ser favorable para


la compaa en base a sus competencias centrales y a las
sinergias que genere. Debe evaluarse si dichas competen-
cias, requeridas para el nuevo negocio, son similares a las que
desarrollaron las compaas que ya estn dentro del mismo.

Consideraciones a tener en cuenta:

Sinergias de ingresos tras la canibalizacin con


productos existentes.
Costos de distribucin y facilidad de acceso a
canales de distribucin.
Costos de capital, dado el riesgo de la inversin.
Potenciales economas de escala y curvas de
aprendizaje, particularmente con respecto a la
competencia.
Costos de la sinergia.
Incremento en la complejidad.
Usos alternativos de la inversin.

Si no se comparte ninguna competencia fundamental, y no


existe sinergia entre los dos negocios, la empresa no tendr
ninguna ventaja competitiva con respecto a la empresa pro-
medio que ya pertenece al nuevo negocio, y probablemente
no deber entrar al mismo. Si existe sinergia, es conveniente
asegurarse de que sta contrarreste los costos complejos
asociados a entrar al nuevo negocio.

Es importante considerar el tema cultural de las empresas


antes de la fusin ya que puede ser una futura causa de
problemas y fracaso.
47

3.4. Estrategias de Marketing

Se recomienda aplicar este framework de las 4P a preguntas


sobre marketing y sobre el desarrollo de productos nuevos.

Producto
Debe encajar en la decisin de posicionamiento y
de segmentacin
Bienes diferenciados vs. Commodities.
Caractersticas y capacidades.
Confiabilidad, calidad, marca, reputacin.
Packaging, tamao.
Servicio, garantas.
Estrategia futura para el producto.

Plaza (Distribucin)
Canales de distribucin (en funcin de las
especificaciones del producto, el nivel de control
deseado y los mrgenes buscados)
Cobertura (equilibrio entre niveles de cobertura
buscados y costos de la misma)
Inventario (nivel, rotacin, costos de almacenamiento)
Alternativas de transporte, eficiencias, costos de
transporte.

Promocin

Proceso de compra:
Conocimiento del producto por parte del cliente
Inters del cliente por el producto
Perodo de prueba
El cliente lo vuelve a comprar
Lealtad
Seleccin mtodo de venta:
Pull (publicidad, por ejemplo) o Push (descuentos al distri-
buidor, por ejemplo).

Casebook Trama 2015


48

Cinco categoras de esfuerzos de promocin:


Publicidad: medio, alcance (porcin del mercado
meta alcanza) y frecuencia
Ventas personales: particularmente, cuando se
necesita contacto directo con el cliente
Promocin de ventas: incentivos al consumidor, a la
fuerza de ventas y a los miembros del canal:
-Incentivos al consumidor: cupones,
reembolsos, muestras, categoras Premium,
concursos.
-Incentivos a las fuerzas comerciales:
concurso de ventas, carteles en el punto de
venta, reputacin de las franquicias,
demostraciones dentro de la tienda.
Relaciones pblicas y Publicidad
Venta directa

Precio

Considerar precio minorista y descuentos


Qu estrategia utilizar? Costo marginal = Ingreso
marginal, precios altos para obtener ingresos
ahora, precios bajos para ganar market share?
Buscar volumen de ventas, o beneficios?
Valor percibido, o establecimiento del margen
de ganancias como un plus al costo de produccin
(Cost-plus-margin pricing)?
Cmo se relaciona el precio al mercado, al
tamao, al ciclo de vida del producto y a la
competencia?
Incentivos econmicos en el canal (comisiones,
mrgenes)
Establecimiento de barreras de entrada
49

3.5. Mini Frameworks

Interno vs. Externo

Cuando no se est seguro de cul es la estructura ms ade-


cuada para profundizar en algn concepto, es un buen pun-
to de partida separar entre factores internos y externos.

Cuantitativo vs. Cualitativo

Este framework sirve de herramienta para estructurar razo-


nes de decisin y evaluacin. Por ejemplo, un mercado que
cuantitativamente no sea rentable puede que cualitativa-
mente sea muy atractivo, ya que nos permite ser el nexo a
otro mercado muy rentable.

Costo (dificultad) vs. Beneficio (impacto)

Este framework es til para tomar decisiones poniendo en la ba-


lanza los posibles beneficios en contraposicin con los costos
de obtener dichos beneficios. Conviene representar dicho fra-
mework en forma de matriz donde se separen cuadrantes para
las distintas situaciones extremas.

Casebook Trama 2015


50
51

4. Casos de Prctica
Caso N 1: GREEN BEANS NASCAR TEAM

Objetivo

El objetivo del caso a seguir es validar y testear el razonamien-


to lgico del candidato ante problemticas similares a las cua-
les se encontrar en su trabajo como consultor.

Nota para el Entrevistador: Es importante comenzar el caso


indicndole el objetivo al candidato, resaltando que no es parte
fundamental de la resolucin la exactitud numrica como s lo es
el raciocinio y la estructuracin de la respuesta. A la vez, caso
el candidato se encuentre preparado, es recomendable realizar
preguntas adicionales fuera de la estructura del caso para testear
su reaccin.

Contexto

Green Beans es una empresa de bebidas sin alcohol con un


alto foco en la publicidad como parte de su estrategia de co-
municacin y posicionamiento de la marca.

En la actualidad la empresa es duea de un equipo de NAS-


CAR como parte de este objetivo. El NASCAR es una empresa
familiar que gerencia y controla eventos de competiciones de
autos en los Estados Unidos.

Las carreras de NASCAR estn solamente detrs de la NFL


(National Football League) en tamao de audiencia en los EUA
y son transmitidas a ms de 150 pases. Varias Empresas del
Fortune 500, son patrocinadores de esta competicin, aunque
el nmero ha ido reducindose desde los aos 2000.

Casebook Trama 2015


52

Etapa 1: Anlisis de la inversin (15 a 20 minutos)

Pregunta

El contrato con NASCAR est por vencer a fines de este ao


y el CEO de Green Beans solicit nuestros servicios para ayu-
darlo a evaluar la necesidad de permanencia de la marca como
duea de un equipo para el ao siguiente y tomar la decisin
de si vale la pena continuar con esta inversin.

Nota para el Entrevistador: En este punto el candidato debera


haber entendido la problemtica y es libre de realizar preguntas si
lo considera necesario.

Informacin disponible

La Sprint Cup de NASCAR corre un total de 36 carreras por


ao con 40 equipos participantes. Ser el principal sponsor
de uno de estos equipos cuesta entre U$D 350.000 y U$D
500.000 por carrera, el sponsor puede elegir el diseo del
auto, colocar el logo en l, y utilizar a los pilotos en sus pu-
blicidades. El tiempo medio de una carrera es de 2,5hs con
una preparacin inicial de 0,5hs. El tiempo de la ceremonia
de podios es de 15 minutos.

Como resultado de esta inversin se encuentra la posibilidad


de aumentar la base de fans a partir de los 50 millones de tele-
videntes que asisten a cada carrera por los distintos medio de
comunicacin. Algunas empresas calculan el retorno de la in-
versin en base a al nmero de segundos que su logo aparece
en la pantalla, multiplicndolo por la tasa actual de conversin
de televidentes a consumidores de la marca.

A partir de un anlisis realizado sobre informacin de ven-


tas, precios y publicidad de todas las categoras relevantes de
consumo masivo en los Estados Unidos, se obtuvo que un au-
mento de 1 Minuto de tiempo de exposicin en la Televisin,
trae un aumento de ventas de U$D 0,6 millones, aunque vara
por categora y marca.
53

En el ao 2014 Green Beans invirti U$D 100 millones en el


patrocinio total de un equipo, el cual gan 2 carreras y accedi
al podio en otras 4. Green Beans mantuvo como el principal
logo en el diseo del auto, y este tena sus colores basados
en los colores de la empresa. Existe la posibilidad de vender
los espacios del diseo que no fueron ocupados por la em-
presa. En este perodo fueron vendidos a otras empresas por
U$D 50 millones.

Nota para el Entrevistador: La informacin de inversin del pa-


trocinio debe ser entregada al candidato junto con la informacin
de la venta de publicidad para que pueda tener visin de la inver-
sin neta requerida.

Cuadro de Resultados (P&L) 2014:

Fuerza de Ventas: Compuesto por los vendedores y el


Backoffice de soporte de ventas.
Trade Marketing: Compuesto por el Merchandising en
las tiendas.
Distribucin: Compuesto por gastos de entrega y
almacenamiento.
Marketing: Compuestos por el equipo de NASCAR
y otras fuentes de publicidad.
Administrativos: Gastos generales y administrativos.

Casebook Trama 2015


54

Nota para el Entrevistador: Estas informaciones estn disponi-


bles para ser entregadas al candidato en caso de que pregunte
por ellas para la resolucin numrica del problema. En caso de
que el candidato realice suposiciones numricas, est en el crite-
rio del entrevistador corregirlo en base a los nmeros dados.

Resolucin sugerida

Se procede a calcular el valor total de la inversin, a partir de


los U$D 100 millones invertidos durante el 2014, menos el in-
greso por la venta de publicidad a otras empresas de U$D 50
millones, dando una inversin neta de U$D 50 millones anuales.

A continuacin, se procede a calcular el retorno esperado de


esa inversin. Sabemos que para calcular este retorno debe-
mos considerar los segundos de exposicin de la empresa en
la televisin por la tasa de conversin de televidentes. Adems,
se tiene el dato de cul es la ganancia por minuto que Green
Beans obtuvo en el ao 2013, con U$D 600.000 por minuto.

Para calcular los minutos, debemos sumar tanto los minutos


de exposicin durante la carrera como durante la premiacin
en el podio. Por un lado, el tiempo de carrera y de podios que
ser televisado es de 180 y 15 minutos respectivamente. Con-
siderando que ese tiempo se divide de forma proporcional
entre los corredores que participan en cada etapa, existe un
tiempo neto de 180/40 = 4,5 minutos disponibles para cada
corredor en la carrera, y de 15/3 = 5 minutos durante el podio.
Tomando la cantidad de carreras corridas y podios consegui-
dos por el equipo de Green Beans se calcula el total de minutos
de exposicin que tendr el logo de la empresa en la televisin:

Carrera= 4,5 min x 36 carreras = 162 minutos


Podios = 5min x 6 podios = 30 minutos
Total Minutos = 192 minutos

As, estimamos el total de aumento de ventas: 192 minutos x


U$D 0,6M/minuto = U$D M 115,2.
55

Nota para el Entrevistador: Se presenta un rbol con la lgica


sugerida a seguir para el clculo del ingreso. Es muy probable
que el candidato llegue a una estimacin de ventas y no prosiga
con el anlisis de contribucin marginal. En este caso, se deber
conducirlo, a partir de una nueva pregunta, para que realice este
paso adicional y llegue a la conclusin.

Sin embargo, para tomar la decisin sobre la inversin se debe


realizar un anlisis marginal del gasto en el NASCAR, calculan-
do el cambio en ingresos provocado por el cambio en ventas.
A partir del cuadro de resultados podemos ver que como gas-
tos variables Green Beans posee el costo de produccin y el
de distribucin, sumando un 59% de la facturacin. De esta
forma obtenemos el delta de resultados si las ventas aumentan
en lo estimado: U$D M 115,2 x 41% = U$D M 47,3.
Con este resultado, el gasto de U$D M 50 no tiene retorno de
ingresos con lo cual no sera recomendable seguir con la inver-
sin. Sin embargo, vale destacar que esta conclusin depende
fuertemente del nmero de podios que se obtengan, dado que
con 9 podios la inversin en el NASCAR ya se repagara.

Casebook Trama 2015


56

Evaluacin del candidato

El candidato debera ser capaz de identificar una estructura


para el clculo de aumento de ventas en base a los minutos
de exposicin de la marca, y establecer un racional lgico de
dimensionamiento de esos valores.

Por otro lado, agrega valor a la respuesta si el candidato en-


tiende por s mismo que el cambio en ventas no es suficiente
para analizar la inversin, sino que debe ser entendido el im-
pacto en los ingresos que este trae. Solamente con el anlisis
marginal es que se puede llegar a la conclusin acertada del
retorno del dinero invertido.

Finalmente, una respuesta ptima incluira identificar que exis-


ten dos momentos diferenciados de exposicin, tanto en la
carrera como en el podio, agrega valor a la respuesta y que el
nmero de podios es una variable clave en la determinacin

Etapa 2: Usos alternativos del Dinero Disponible


(5 a 10 minutos)

Pregunta

En base a la situacin actual en que se encuentra la compaa,


el CEO de Green Beans solicita un anlisis adicional para iden-
tificar si existen otros destinos internos para el dinero disponi-
ble que impacten en las ventas y qu uso se le dara en cada
uno de ellos.

Nota para el Entrevistador: El objetivo de esta segunda pregun-


ta es que el candidato realice un anlisis totalmente cualitativo
de usos alternativos del dinero disponible, testeando la visin de
negocios que transmite. A continuacin, se brinda una serie de
informaciones del status actual de la compaa que deben ser
usados como respuesta por el entrevistador, para que el candi-
dato entienda la situacin actual de Green Beans.
57

Informacin disponible

Comercialmente la empresa realiz durante el 2014 un diagns-


tico de su llegada al mercado, descubriendo que su presencia
en la regin central del pas est en deficiencia al comparar con
los principales competidores. Por otro lado, el merchandising
ofrecido slo llega a un 30% de las tiendas en las cules es
comercializado el producto y consta solamente de un artculo
que no ha sufrido cambios en los ltimos 2 aos.
Respecto de la Logstica, las quejas de parte de los consumi-
dores han estado en aumento en los ltimos meses, principal-
mente porque no encuentran el producto en los puntos de ven-
ta donde generalmente los obtenan. Internamente se encontr
que efectivamente los quiebres de stock han aumentado, aun
cuando la demanda se mantuvo estable.
Por ltimo, mientras que los competidores han realizado en
promedio 3 lanzamientos de nuevos productos por ao, Green
Beans mantuvo durante los ltimos dos aos el mismo portfo-
lio sin modificaciones.

Nota para el Entrevistador: Dada que la resolucin es abierta,


la informacin a disposicin muestra los principales caminos que
deberan ser considerados por el candidato. Caso este no se en-
foque en ninguno de estos aspectos, el entrevistador puede rea-
lizar una pregunta especfica sobre alguno de ellos para testear la
reaccin del candidato.

Resolucin

El dinero puede ser utilizado para mejorar los aspectos men-


cionados en los cules la empresa muestra deficiencias:

Fuerza de Ventas, podra ser utilizado para incrementar la


cantidad de vendedores y mejorar la llegada a las regiones
desatendidas.
Trade Marketing, podra invertirse en campaas de con-
tacto con el consumidor, ofreciendo nuevas formas de ex-
hibir el producto y aumentando el nmero de tiendas donde

Casebook Trama 2015


58

se ofrece merchandising.
Logstica, podra invertirse en el aumento de la flota de ca-
miones disponible y en aumento de stock en los centros de
distribucin, para asegurar mejores niveles de servicio evitan-
do los quiebres en las tiendas.
I+D, podra invertirse en el desarrollo de nuevos productos, a
partir de investigaciones de mercado y benchmarks para cap-
tar las nuevas tendencias de consumo.

Evaluacin del Candidato

El candidato debera ser capaz de identificar que para un au-


mento de ventas puede enfocarse la inversin tanto en accio-
nes de Marketing, como el aumento de la fuerza de ventas o
gastos de trade marketing, en acciones de, incrementando la
flota de distribucin o el almacenaje para disponibilidad pro-
ducto a ms clientes y de mejor forma o en acciones de I+D
para actualizar el portfolio a las nuevas tendencias. Si adems
el candidato logra identificar cmo invertira ese dinero, mos-
trando una visin de negocios coherente y racional, agrega va-
lor a su respuesta.

Caso N 2: Banco Alfa

Escenario Inicial

Usted se desempea como lder de proyecto en una reconocida


consultora que asesora a empresas pertenecientes a la indus-
tria de servicios financieros. Uno de los principales clientes de la
compaa, el Banco Alfa, le solicita sus servicios debido a que ha
registrado que en los ltimos 6 meses la rentabilidad promedio
por cliente disminuy 3 puntos porcentuales a pesar de que el
aumento de la cantidad total de clientes en el mismo perodo fue
del 5%. El crecimiento semestral de la cartera de clientes fue de
7% en los ltimos dos aos con excepcin del ltimo perodo.
Adems, en el ltimo ao el banco ha padecido una merma en su
posicionamiento en el mercado ocupando el 5 puesto.
59

El banco le solicita que disee un plan de accin a 3 meses


para recuperar el posicionamiento perdido en el mercado, me-
jorar la captacin y retencin de clientes a los cuales el banco
quiere enfocarse.

Cules son las consultas que usted le formulara al cliente


para dimensionar correctamente la problemtica, y comenzar
a analizar y a ejecutar el proyecto?

Qu plan de accin recomendara?

Preguntas deseables

P: Cules son los objetivos concretos que se quieren


alcanzar con esta iniciativa?
R: Se pretende aumentar la captacin y retencin de
clientes de renta alta (que son aquellos con mayores
ingresos estimados), posicionarse en el mercado
entre los 3 primeros bancos para ese segmento.

P: Cmo es la composicin actual de la cartera de


clientes del banco? Cunto representa en ingresos Esta tabla muestra
que el segmento
cada uno? de Baja/Media Baja
R: Los clientes se clasifican en segmentos en funcin y Media represen-
ta el 80% de los
de sus ingresos estimados. La cartera de clientes se clientes.
compone de la siguiente manera: Rentas

Casebook Trama 2015


60

P: Cul es la tendencia en el mercado? Ha variado


la demanda? La competencia tambin ha sufrido
disminucin de su cartera de clientes?
R: La demanda no ha variado en los ltimos 6 meses,
pero la participacin del banco en el mercado
ha disminuido.

P: Cules son los indicadores en los que se basan


para definir que el banco perdi clientes/posiciona-
miento/rentabilidad por cliente? Cul es su forma de
medicin y actualizacin?
R: La cartera se mide por la cantidad de clientes por
segmento registrados en el sistema comercial.

El posicionamiento se estima en funcin de los resultados de


encuestas de satisfaccin que se realizan semestralmente a
una muestra representativa de clientes y prospectos (poten-
ciales clientes), mediante el uso de distintos canales de co-
municacin. Las encuestas indican que las personas destacan
como aspectos positivos del banco el trato y cordialidad de su
personal y la accesibilidad de informacin a travs del Home-
banking, como aspectos negativos percibidos sobresalen los
tiempos de espera en sucursales y la falta de rapidez y eficien-
cia a la hora de solucionar problemas y/o realizar trmites.

Los ingresos estimados se calculan en funcin a ciertas carac-


tersticas del cliente, y la rentabilidad promedio es la suma de los
ingresos estimados dividido por la cantidad total de clientes.

P: Qu acciones recientes relevantes ha hecho la


competencia?
R: La competencia no ha realizado acciones
relevantes en los ltimos meses.

P: Cules son los canales de atencin con los que


cuenta el banco hoy en da?
R: Canales de atencin convencionales: Atencin
personal en sucursales.
61

Canales alternativos: ATM (Cajeros automticos),


Homebanking, Telfono, Telefona mvil.

P: Cunta de la capacidad mxima de atencin de


los canales se utiliza?
R: La capacidad de atencin de ejecutivos de
cuenta (personas que atienden personalmente a
clientes en sucursales) se encuentra desbordada, y
no se dispone de espacio fsico en sucursales para
aumentar la cantidad de puestos de ejecutivos de
cuenta. Se estima que entre el 20% y 30% de las
operaciones podran derivarse a canales actuales
de autogestin.

P: En los ltimos 2 aos, Han variado los productos


y/o servicios que ofrece el banco a sus clientes?
R: No

P: En los ltimos 2 aos, Ha variado el precio los


productos y/o servicios que ofrece el banco a sus
clientes?
R: Los precios (en pesos) han acompaado el
aumento de precios por inflacin, siendo precios
competitivos en el mercado.

P: Cules son los beneficios del cliente en funcin


del segmento al que pertenecen?
R: En funcin al segmento al que pertenecen, los
clientes tienen acceso a distintas lneas de productos
y servicios. Los clientes de Renta Baja/Media Baja
acceden a productos y servicios bsicos, mientras
que los clientes con mejor renta pueden acceder
adems a un paquete de productos superiores.
El servicio de atencin es el mismo para todos
los segmentos.

Casebook Trama 2015


62

Recomendaciones deseables

Para captar y retener clientes de Renta Alta:

Diferenciar el servicio de atencin a clientes preferenciales


con canales de atencin exclusivas.
Definir estndares de calidad y tiempos de procesos a
cumplir para mejorar el nivel de atencin a este segmento
de clientes.
Implementar reglas de prioridad en canales de atencin y en
procesos en funcin del segmento al que pertenece el cliente.
Disponer de ejecutivos de cuenta exclusivos para la aten-
cin de este tipo de clientes.
Disear y ofrecer descuentos y beneficios de acuerdo al
perfil representativo del cliente de cada segmento.

Para mejorar la calidad de atencin al cliente:

Incorporar canales de autogestin.


Aumentar la derivacin de operaciones a canales de autogestin.
Incorporar tecnologa para automatizacin de trmites.

Nuevo escenario

Como parte del plan de mejora del servicio de atencin al


cliente, el banco ha decidido adquirir equipos (ATM y Ter-
minales autoservicio) a modo de aumentar la capacidad de
sus canales de autogestin.

El equipamiento ha sido encargado a una empresa europea


al comenzar el proyecto, y el proveedor ha prometido entre-
gar los equipos en el plazo de un mes. De acuerdo al plan de
proyecto, a los 12 meses de iniciado el proyecto se deber
mostrar al directorio de Casa Central el impacto de la imple-
mentacin de los nuevos equipos sobre los indicadores de
calidad de servicio.

Al momento de cumplirse 3 semanas de ejecucin del pro-


yecto, el gobierno emiti una nueva resolucin aduanera por
63

la que los equipos quedarn retenidos en aduana en un pla-


zo que se estima ser de 6 meses.

Dado que, en funcin de los indicadores que se muestren en


la reunin de directorio planificada para el mes 9 se decidir
si el proyecto tendr continuidad o no, el banco le solicita
ayuda para disear un plan alternativo que permita mejorar
la calidad de atencin al cliente hasta tanto no se pueda
hacer uso de los nuevos equipos.

Qu plan de accin alternativo recomendara al banco?

Recomendaciones deseables

Fomentar el uso de canales de autogestin, de modo que


se aproveche al mximo la capacidad actual disponible de
estos canales, para lo cual se recomienda realizar campa-
as informativas y formativas acerca del uso de los equipos
y sus beneficios (va mail, banners, folletera, publicidad en
medios de comunicacin, etc.), y disponer de personal que
asista a los usuarios de las terminales de autogestin.

Competencias deseables de un consultor

A continuacin compartimos algunos ejes de evaluacin


que el entrevistador deber atender a partir de este caso.

Actitud: se evala la capacidad para afrontar desafos, el r-


pido set-up, flexibilidad, autonoma, adaptacin, capacidad
de escucha activa, empata, entusiasmo.

Comunicacin: utilizacin de estrategias efectivas, claridad


conceptual, balance entre la sntesis y el detalle, influencia.

Resolucin de problemas y toma de decisiones: concepcin del


problema, capacidad de anlisis, estrategias simultneas, m-
todos y herramientas efectivos, pensamiento crtico, creativi-
dad, innovacin y visin sistmica.

Casebook Trama 2015


64

Orientacin al cliente y a los resultados: entendimiento del nego-


cio del cliente, propuestas concretas que excedan expecta-
tivas y a partir de un correcto entendimiento de necesida-
des.

Conocimientos tcnico metodolgicos, funcionales y de industria:


basamento de las soluciones en conocimientos aplicables a
la realidad del cliente.

Caso N 3: Starbucks Coffee

Introduccin

Nuestro cliente es Starbucks, empresa global que vende


caf, entre otros productos alimenticios, en distintos tama-
os y sabores. Su Gerente General para Argentina, Jorge
Schultz, nos contact luego de haber identificado un pro-
blema al que pretende solucionar. Jorge nos cuenta que
Starbucks comienza sus operaciones en Argentina hace tan
slo 5 aos, y en este tiempo ha logrado abrir 50 locales (to-
dos en grandes ciudades del pas). En el ltimo ao, Jorge
participa de una revisin global donde reporta su performan-
ce a Global y recibe feedback que sus ganancias netas de
bebidas con cafena (Caf y T) en Argentina son menores a
las esperadas. Es por este motivo que nos contacta y nos
pide nuestra ayuda: Quiere entender las posibles causas y
posibles soluciones de este problema.

Framework esperado y preguntas direccionales

En caso que el entrevistado no comience a realizar pregun-


tas para aclarar conceptos, el entrevistador puede comen-
zar con la siguiente pregunta como gua:

Entrevistador: Qu reas o aspectos se deberan


estudiar para obtener una respuesta al
65

problema planteado?

Entrevistado: Puede escribir sobre un papel o no


el Profitability Framework (Ingresos y costos), pero
debe tener esto en la cabeza para comenzar a
analizar las distintas reas.

En caso que el entrevistado no comience a realizar pregun-


tas para aclarar conceptos, el entrevistador puede comen-
zar con la siguiente aclaracin como gua:

Entrevistador: Starbucks se encuentra con una


gran duda: si el problema reside en volmenes de
ventas menores a los esperados o en menores
rentabilidades

Es necesario que el entrevistado entienda los componentes


que componen a las ganancias de una empresa. Por lo tan-
to, el entrevistador puede optar por preguntar al entrevista-
do para forzar la estructura del caso:

Entrevistador:Cules son los componentes de la


ganancia neta de una empresa? Podras escribir
la ecuacin?

Casebook Trama 2015


66

Entrevistador: Cmo podemos saber si la cantidad


de ventas de Starbucks est en lnea con la
esperada? Podemos preguntarle al gerente de
ventas la cantidad de cafs que venden, pero cmo
sabemos si esa cantidad es acorde a lo esperado
para Starbucks en Argentina?

El entrevistado podra sugerir compararnos con el mercado to-


tal. Para eso, podra proponer adquirir informacin que releve
ventas del mercado (por ejemplo, Nielsen) y comparar nuestro
Share con el resto de los competidores. Sin embargo, estos
datos por s solos podran no ser concluyentes. Por ejemplo,
si los datos nos dicen que Starbucks vende el 5% de los cafs
consumidos en Argentina, podemos concluir que vende por
debajo de lo esperado? Es lo mismo si Starbucks tiene 1 slo
local o 30 locales? Es lo mismo si los cafs se consumen en
todo el pas o si el 90% del volumen se consume slo en Bue-
nos Aires (donde Starbucks tiene sus locales)? Es lo mismo si
el 90% de los cafs consumidos son cafs pequeos, simples
(cafs negros), consumidos en cafeteras de barrio?

Entrevistador: Ms all de la informacin del mercado,


sabiendo que tenemos 20 locales, qu tipo de
anlisis se te ocurre que podramos hacer, para ver
si estamos vendiendo la cantidad de cafs que
deberamos vender?

Entrevistado: Podramos estimar las ventas que


podramos hacer (si los locales trabajaran al 80-
90% de su capacidad) y comparar ese dato con las
ventas reales que hacemos. Si vemos que coinciden
(o que estn cerca), significa que estamos vendiendo
la cantidad de cafs esperados, y por lo tanto el
problema est en otra rea.
67

Aclaracin: cualquiera de estas metodologas es vlida,


siempre y cuando todos los supuestos tenidos en cuenta
estn respaldados por una lgica razonable. La metodologa
esperada es la que presenta menos dificultades a la hora de
pensar los supuestos.

Antes de profundizar en el dimensionamiento, el entrevista-


do debera realizar una estructura cualitativa de la metodo-
loga a emplear, como se muestra a continuacin:

Entrevistado: Voy a estimar la cantidad de horas


que se encuentra abierto Starbucks, luego la
cantidad de vasos de caf que se venden por hora
para obtener las ventas diarias de cada local.
Te parece correcta esta metodologa?

Se muestra a continuacin una tabla con los supuestos razo-


nables a tener en cuenta para resolver el dimensionamiento

Casebook Trama 2015


68

Atencin: Si bien los datos son Assumptions, es importante


que sean razonables. Por ejemplo, si el entrevistado estima
que Starbucks est abierto 8 horas por da, es necesario que
el entrevistador lo interpele y pregunte: A qu hora abre y
cierra?. Si estima slo 8 horas, por definicin, el entrevis-
tado est asumiendo que los locales estn cerrado a la hora
del desayuno, la del almuerzo, o la de la tarde (una asuncin
no razonable que merece ser ajustada).
69

Otros aspectos extra a considerar para mejorar el


dimensionamiento

Considerar que la demanda puede variar por local (no de-


bera ser lo mismo un local en el Microcentro que un shop-
ping).
Nota: No es necesario realizar el impacto cuantitativo de estos
supuestos, pero s sera importante identificar estos puntos.

Otros supuestos extra (hora pico vs no pico, tener en


cuenta los feriados)

Es importante explicar la lgica a la hora de realizar los su-


puestos (por ejemplo, por qu se consideran 2 rdenes por
minuto? Porque el tiempo promedio para cobrar un pedido
se estima en 30 segundos)

No es relevante el nmero final obtenido, pero s la meto-


dologa y razonamiento empleado. El entrevistador, en caso
que el nmero sea demasiado alto se lo comunicar al en-
trevistado (identificando el supuesto con el mayor desvo
frente al esperado)

El Entrevistador debe plantear el siguiente punto

Entrevistador: Starbucks vende 90 M de cafs en


Argentina. Qu podemos concluir entonces?

Entrevistado: Si bien es ligeramente menor al


esperado, est muy cerca en cunto a rdenes de
magnitud, por lo que podramos concluir que el
volumen vendido es razonable y no pareciera ser la
raz del problemade la rentabilidad. Deberamos
buscar en otras reas. Por ejemplo, podramos
investigar si los precios o los costos estn en los
niveles esperados

Entrevistador: Estoy de acuerdo. Pidmosle al CFO


de la empresa algn resumen de los precios y costos,
e investiguemos qu se esconde detrs de estos nmeros

Casebook Trama 2015


70

Evaluacin de potencial problema de precios y costos

A continuacin, el entrevistador debe presentar las siguien-


tes tablas para posterior anlisis
71

Luego de presentadas las tablas, el entrevistado debe to-


marse un tiempo para analizar las tablas y poder concluir
que las mayores ventas de Starbucks estn en productos de
menores ganancias netas. Por ejemplo, el 45% de los vasos
vendidos son de Regular o Medium, producto que tiene una
ganancia de slo $4, en contraste con los vasos Specialty,
que cada uno representa slo entre el 2% y el 10% del vo-
lumen, aunque otorgan una ganancia de entre $9.50 y $13
por vaso.

Atencin: El anlisis debe hacer slo el valor absoluto de


ganancias y no del valor relativo (%). Por ejemplo, el entre-
vistador podra plantear un producto que tenga un precio de
venta de $1 y un costo de $0.1. Por lo tanto tiene un mar-
gen de ganancia de 90%. Es preferible vender un vaso de
este producto, o de cualquier otro de nuestro portfolio? La

Casebook Trama 2015


72

respuesta SIEMPRE es cualquier otro de nuestro portfolio,


dado que este vaso nos otorga $0.9 por vaso vendido, mien-
tras que cualquier otro producto nos dar $3 o ms.

Pensamiento estratgico e idea de negocios - Iniciativas

El entrevistador debe guiar al entrevistado con la siguiente frase

Entrevistador: Qu se te ocurre que podemos


sugerir (estrategias) para resolver este problema?
Cules seran las distintas iniciativas a implementar
con el objetivo de mejorar la rentabilidad de la
empresa? Cules seran las desventajas de las
mismas? Cul sera la ms recomendable
para Starbucks?

Respuestas esperadas

Reducir costos

Descripcin: De produccin, costos fijos

Desventajas: Dificultad para bajar los costos an ms que en


la actualidad

Impacto: 10% de ganancia mxima, por ejemplo.

Preguntas para guiar el pensamiento: En qu porcentaje piensa


que se pueden reducir los costos? Qu tan fcil es esto?
En qu reas de la empresa se puede llevar a cabo esta re-
duccin? No crees que una empresa de las dimensiones de
Starbucks podra haber identificado esto sin la necesidad
de contratarnos?

Incrementar precios

Descripcin: En todos los locales, en algunas regiones, etc.

Desventajas: Posibilidad de perder clientes


73

Impacto: Se necesita un anlisis de sensibilidad de demanda


en funcin del precio

Preguntas para guiar el pensamiento: Sern los clientes sensibles


al precio del producto? En qu medida? En qu porcentaje se
podra aumentar el precio?

Cambiar el mix de productos

Descripcin: Promociones, incrementar precio del Vaso Regu-


lar, marketing, bajar precio de Specialty, hacer nuevas frmu-
las, entender por qu no pueden vender ms volumen (a los
clientes no les gusta el producto, es muy caro, etc.)

Desventajas: Dependiendo de la estrategia elegida para cam-


biar el mix de productos, cambiaran las barreras que podran
llegar a surgir.

Impacto: Alto

Preguntas para guiar el pensamiento: Cmo podra variar el mix


por ubicacin del punto de venta? Como podra variar por tipo
de cliente (quin compra el Specialty)? Qu tipo de acciones
promocionales se pueden realizar (combos, descuentos)?

Por ltimo, el entrevistador debe buscar que el entrevistado


termine de resolver su caso, con la siguiente frase:

Entrevistador: Para terminar, por qu piensas que la


performance real de Starbucks en Argentina no estuvo
alineada con las ideas originales de Global? Qu
explicara las ventas actuales de Starbucks?

No se realiz un buen estudio cultural, y se asumi que el mix


de productos de otro pas donde Starbucks cuenta con una
buena performance (por ejemplo, Estados Unidos), se iba a
cumplir en Argentina.

Existe una preferencia cultural en Argentina hacia otros tipos


de caf no tan sofisticados (en el caso que el entrevistado haya

Casebook Trama 2015


74

identificado lo de cambiar el mix de productos).

Preferencias culturales hacia tamaos de caf pequeos.

Estructura y Sntesis - Resumen y conclusiones

Entrevistador: Durante una visita al cliente, te


encontrs con el Gerente de Innovacin y te pide que
le cuentes el anlisis y resultados en 30 segundos.

Entrevistado: Al haber identificado bajas ganancias


netas, se decidi hacer distintos anlisis para primero
identificar la problemtica. En primer lugar se estim
el volumen esperado de ventas y result coherente
con el real, por lo que se procedi a analizar en
profundidad los costos, precios y ganancias de cada
SKU. Se pudo identificar que el problema reside en
el mix de los productos ofrecidos, por lo que se
recomienda incentivar la migracin de clientes desde
el Caf Regular hacia el Specialty mediante Iniciativa
A y B.

Caso N 4: Autopartista

Introduccin

Nuestro cliente es una autopartista global que tiene ofici-


nas centrales en Estados Unidos. Su negocio principal, con
20% de participacin en el mercado, incluye unos 5,000
SKU diferentes. Hace varios aos que el negocio no es
rentable y el presidente de la compaa sospecha que su
alto grado de integracin vertical los est desfavorecien-
do: produce en plantas propias alrededor del 80% de sus
SKUs, comparado con 40% de los principales competi-
dores. El presidente pidi nuestra ayuda. De qu manera
enfrentaras el problema?
75

Gua para el entrevistador

Evaluar si el entrevistado cubre los puntos clave

Por qu el negocio ha sido poco rentable en los ltimos aos?


Cmo y cunto contribuye a la baja rentabilidad el nivel
de integracin actual?

Informar si lo preguntan (datos disponibles)


La compaa divide sus productos en tres segmentos: A,
B y C, segn el precio medio del rea, A=$30; B=$10; C=$8
El mix de productos ofrecido (SKU por segmento) es:
A=10%; B=40% y C=50%
Volumen de venta anual: A=1,000,000; B=3,000,000;
C=200,000 unidades
Costos de fabricacin interna por categora: A=$12,000,000;
B=$25,000,000;C=$1,500,000
Costos de adquisicin a terceros por categora: A=$8,000,000;
B=$5,000,000
Volumen de fabricacin en planta propia: A=600,000 de
los productos; B=2,500,000 productos; C=100% de
los productos.

Posible resolucin preguntas del entrevistado

Entrevistado: Con respecto al caso lo dividira en dos


secciones. Primero me gustara evaluar la baja
rentabilidad y luego entender hasta qu punto esta
baja en rentabilidad est conectadacon la alta
integracin vertical de la compaa. Con respecto a
la rentabilidad, me interesa analizar las tendencias
en costos, precios y contribucin marginal, si es
posible por producto o por segmento de productos.

Entrevistador: Tenemos informacin sobre el precio


de cada producto, la compaa los divide en tres
grandes grupos: el grupo A tienen un precio medio
de $30; el grupo B de $10 y el grupoC de $8.

Casebook Trama 2015


76

Entrevistado: Me interesa saber el impacto sobre


la ganancia que tiene cada categora de producto,
para ello necesito saber la cantidad vendida de
cada producto se tiene dicha informacin?

Entrevistador: Esa informacin est disponible


nuevamente por categora de productos. Se
puede aproximar que en cada categora las ventas
se distribuyen casi homogneamente. Para la
categora A se venden 1,000,000 de productos por
ao, para la categora B 3,000,000 de productos por
ao y para la categora C 200,000 de productos por
ao. Qu conclusin podras sacar de la
informacin brindada?

Entrevistado: La facturacin anual por categora de


producto es:
o 1,000,000_ $30 = $30,000,000
o 3,000,000_ $10 = $30,000,000
o 200,000_ $8 = $1,600,000

Se ve una clara diferencia entre los segmentos: las catego-


ras A y B lideran en facturacin con un poco menos de 49%
cada una, mientras que la categora C tiene menos del 3%.
Volviendo a la cuestin inicial y para terminar de entender la
rentabilidad, me interesa conocer los costos de fabricacin
y de compra de productos a terceros.

Entrevistador: Los costos de fabricacin propia


o interna estn segmentados por categoras
de la misma manera. El costo total para la categora
A es $12,000,000; para la categora B es $25,000,000
y para la categora C es $1,500,000. Los costos de
adquisicin a terceros de los productos del
segmento A son $8,000,000 y $5,000,000 para el B.

Entrevistado: En tal sentido, los ingresos de las


categoras A, B y C superan sus costos de fabricacin,
teniendo as rentabilidad positiva. En particular:
77

La categora C es la de menor rentabilidad y es 11 p.p. me-


nos rentable que la categora siguiente (B).

Con el fin de mejorar la rentabilidad me gustara saber las


cantidades producidas en planta propia de cada segmento
de productos.

Entrevistador: Todos los productos de la categora C


se fabrican internamente, de los productos solo
comercializados, 400,000 son del grupo A y 450,000
son del grupo B.

Entrevistado: Podemos calcular el costo por


producto producido internamente

Y el costo de compra de productos a terceros

Casebook Trama 2015


78

(Solo analizar este indicador puede traer conclusiones erra-


das, hace falta calcular la contribucin y comparar los seg-
mentos para hacer un anlisis exhaustivo. Los resultados se
encuentran en el anlisis anterior: A=33%; B=17%; C=6%)

Cmo es el mix de productos ofrecido dividido por segmentos?

Entrevistador: De los 5,000 SKUs que producimos,


el 10% pertenece al grupo A y 40%pertenece al
grupo B.

Entrevistado: Voy a calcular la cantidad de SKUs en


cada categora para luego entender lafuente de
ingresos de la compaa:

o 10% _ 5,000 = 500 productos de categora A


o 40% _ 5,000 = 2,000 productos de categora B
o 50% _ 5,000 = 2,500 productos de categora C

Los productos C, aparte de tener el menor margen general


y la menor cantidad de ventas, solo se vende, en promedio,
diez veces cada producto de la categora al ao.

Para ver hasta qu punto la alta integracin es consecuen-


cia del mal desempeo del segmento C, necesito ms infor-
macin de los componentes del costo de fabricacin y as
entender en panorama completo.

Entrevistador: En este momento no tenemos ms


informacin. Cmo pensas que la alta integracin
vertical est afectando la rentabilidad de la empresa?
Hay evidencia con lo visto hasta ahora? Cul es
tu recomendacin con la informacin disponible?

Entrevistado: Si slo consideramos el segmento C,


que es el mayor damnificado, sacamos las
siguientes conclusiones
79

La cantidad vendida es muy poca para la cantidad de


SKUs del segmento (10 ventas promedio al ao por pro-
ducto). Siendo ste, el primer sospechoso aparejado a los
altos costos de fabricacin por producto (conectado a eco-
nomas de escala). En tal sentido hay que entender cul es
la razn de la bajas ventas e intentar mitigarlo si esposible.

Con respecto a los altos costos de fabricacin, se po-


dra evaluar la alternativa de fabricar, parcial o totalmente,
los productos del segmento C externamente. Al conocerlos
costos externos, podremos comparar si nuestros costos de
fabricacin para dicho segmento son competitivos o no, y en-
tender si nuestro problema es realmente la integracin vertical.

El bajo desempeo es atribuible a los productos tipo C.


Las causas pareceran ser:

o Baja eficiencia productiva, que en parte podra


deberse al bajo volumen por SKU.
o Ineficiencia de fabricacin interna por la baja
escala, contra alternativas decomprarlo a
proveedores de mayor volumen y menor costo.

Por otro lado, este segmento representa solamente el 3%


de los ingresos. En tal sentido, las opciones podran ser:

o Reducir significativamente el mix y nmero de SKU,


y mantener slo aquellos que son rentables.
o Externalizar el suministro del resto de los productos
(si el costo lo justifica).
o Debe evaluarse tambin si con la nueva escala de
fabricacin propia (menor a la actual), las plantas
son viables, y reducir o discontinuar aquellas inviables

Si estas no son una posibilidad y si no es un valor estrat-


gico para la compaa, eliminara el segmento C de produc-
cin.

Casebook Trama 2015


80

Evaluacin de la resolucin del caso

Buena

Es necesario hacer un planteo amplio de todos los dri-


vers de rentabilidad, entre otros (si bien no todos son perti-
nentesal caso, hace falta mencionarlos y descartarlos)

- Precios, costos y contribucin marginal por producto;


- Posicionamiento en precios vs. competidores;
brechas y sus causas (posicionamiento, fortaleza
de marcas,capacidad tecnolgica del producto, etc.)
- Mix de productos, volumen y escala para
el abastecimiento;
- Competitividad en calidad y tecnologa;
- Benchmarking contra competidores

Deben considerarse todas las brechas de competitividad


del planteo previo, por ejemplo

- Brecha en costo unitario si se compite con


producto similar
- Brecha en precio si por no contar con diferencias
tales como posicionamiento de marca, capacidad
tecnolgica o de calidad, no se consigue el mismo
posicionamiento y evaluar: Cmo y cunto afecta
el origen de la provisin (proveedores) en dichos
tipos de brecha?

En todo el anlisis, es relevante plantear una mirada segn


mix de productos: las contribuciones o brechas ocurren
con todos los productos o slo con un porcentaje de ellos?

Proponer recomendaciones slidas y seguras, explicando


que tericamente hay que deshacerse del segmento C

Destacada

Para articular la conclusin y recomendaciones, debe


seguirse la secuencia lgica de responder el primer punto
81

(causas de baja rentabilidad) y luego el segundo (incidencia


de la integracin vertical), en la medida en que sea relevante
en su impacto en rentabilidad

El entrevistado sugiere posibles recomendaciones inda-


gando an ms sobre detalles del segmento con menor ren-
tabilidad (valor estratgico, opciones de suministro, volu-
men por SKU y potencial de mejora de costos, entre otros).

Caso N 5: Licitacin de semforos

Introduccin

Nuestro cliente es una empresa que se dedica al negocio de


semforos y se encarga particularmente de las operaciones
de instalacin y mantenimiento pero no de la produccin.
Hoy en da cuenta con la concesin del gobierno para tra-
bajar en la Costa Oeste de Estados Unidos en el Estado de
California. La compaa est evaluando entrar en el nego-
cio de Nueva York, en particular en la isla de Manhattan. El
CEO pidi nuestra ayuda. De qu manera enfrentaras el
problema para analizar la conveniencia de entrar al nuevo
mercado?

Gua para el entrevistador

Evaluar si el entrevistado cubre los puntos clave

Cules son las oportunidades y los riesgos de entrar en


el nuevo mercado?
Cmo es el proceso para adquirir la licencia para operar
en NY? Cunto est dispuesto a ofrecer en la licitacin?

Informar si lo preguntan (datos disponibles)

El perodo de la licitacin en NY es de 3 aos


Monto que paga el Estado a la empresa por instalacin/

Casebook Trama 2015


82

reposicin de un juego de semforos nuevo es $100,000


Monto que paga el Estado a la empresa por mantenimien-
to de un juego de semforos es $15,000
Tasa de reposicin de semforos por ao es de 5%
Mantenimiento se realiza a todos los semforos 1 vez
por ao
Los costos de reposicin/instalacin se dividen en equipo
(semforo) y mano de obra

El equipo le cuesta a la empresa $85,000


La mano de obra sale 35$/h y se requieren 6
personas por 5 horas por juego de semforos para
la instalacin

Para los costos de mantenimiento solo se considera al de


mano de obra

Se requieren 4 personas por 4 horas a un costo de 35$/h

Para estimar la cantidad de semforos sugerir utilizar una


superficie rectangular de 20 calles por 100 calles dando
como resultado 2,000 juegos de semforos. Se asume que
hay un juego de semforos por interseccin. Si el entrevis-
tado obtiene otro nmero sugerir usar 2,000 por simplicidad
de clculo para los siguientes anlisis

Posible resolucin preguntas del entrevistado

Entrevistado: Para estudiar la posible entrada al


mercado de NY analizara por un lado las
oportunidades y por otro lado los posibles riesgos.
En cuanto a las oportunidades, pueden ser a partir
de objetivos estratgicos como la expansin
territorial que luego permita a la compaa crecer
en el negocio de la Costa Este, u objetivos
econmicos como aumentar los beneficios de la
empresa. Los posibles riesgos pueden estar
relacionados con la operacin, por un lado
debido a la lejana con respecto al negocio actual
83

resultando en mayor dificultad de manejo del


negocio y por otro lado por la necesidad de
mayores recursos para poder llevar a cabo
ambos negocios.

Entrevistador: Considerando el punto de vista


econmico, cmo crees que se llega a obtener el
contrato con el gobierno?

Entrevistado: Por tratarse de negocios con el


gobierno de un Estado, el mtodo para obtener la
concesin debera ser a travs de una licitacin
pblica en donde la compaa propone una oferta
para ganarla.

Entrevistador: Efectivamente, se trata de una licitacin


completa en donde la empresa que obtenga la
concesin debe encargarse tanto de la instalacin y
del mantenimiento de los semforos por un perodo
de 3 aos. La pregunta que le surge al cliente es cul
sera el valor mximo que estara dispuesto a ofrecer
la compaa en la licitacin?

Entrevistado: Para conocer el monto mximo a


pagar habra que calcular los beneficios posibles
conociendo tanto futuros ingresos y sus respecti
vos costos. Para calcular los ingresos deberamos
estimar cunto le pagara el Estado a la
compaa por los servicios, tanto de instalacin
como de mantenimiento y luego conocer los
costos asociados.

Entrevistador: Se sabe que el Estado paga $100,000


por cada juego de semforos instalado y $15,000
por el mantenimiento de cada juego. En cuanto a los
costos cmo penss que estn estructurados?

Entrevistado: Para costos deberamos conocer los

Casebook Trama 2015


84

de la compaa considerando los insumos


materiales y la mano de obra tanto para la
instalacin como para el mantenimiento.

Tenemos informacin de los mismos?

Entrevistador: Efectivamente, el costo se divide en


equipos (juego de semforos) y mano de obra
necesaria. Para el caso de la reposicin, el costo
de un semforo es de $85,000 y requiere 6 personas
por un perodo de 5 horas para la instalacin de un
equipo a un costo de $35 por hora. Para el
mantenimiento slo se necesita mano de obra, ms
especficamente 4 persona que trabajen por un
perodo de 4 horas por juego de semforos
remunerados tambin 35$/h.

Entrevistado: Para poder calcular tanto los ingresos


como los costos es necesario conocer la
cantidad de semforos de Manhattan. La empresa
conoce ese dato?

Entrevistador: No se conoce el dato precisamente.


Quisiera que estimes el nmero de juegos de semforos.

Entrevistado: Supongamos que Manhattan se


puede modelizar como una regin rectangular de
20 calles por 100 calles dando un total de
20x100=2000 intersecciones. Tomando que hay un
juego de semforos por interseccin, se llega al
nmero de 2000 semforos. Necesitara saber ade
ms cuntos semforos se instalan por ao y cmo
funciona el mantenimiento para estimar el beneficio
total por ao.

Entrevistador: Estos datos se conocen por la


empresa. Estadsticamente rompe el 5% de los
juegos de semforos por ao y el mantenimiento se
85

le realiza a la totalidad de los semforos una vez


por ao.

Entrevistado: Podemos obtener entonces el


beneficio anual de la siguiente manera

Ingresos:
Instalacin: $100,000 x 5% x 2,000 juegos/ao = $100,000
x 100 juegos = $10 millones/ao
Mantenimiento: $15,000 x 2,000 juegos/ao =$30 millo-
nes/ao
Total: $40 millones/ao x 3 aos = $120 millones

Costos:
Instalacin

Equipos: $85,000 x 5% x 2000 juegos/ao = $85,000 x 100


juegos/ao = $8.5 millones/ao
Mano de obra: 6 personas x 5 horas x 35$/h x 5% x 2000
juegos/ao = $1,050 x 100 juegos/ao = $ 105,000/ao
Total de instalacin: $8.6 millones/ao

Mantenimiento

Mano de obra: 4 personas x 4 horas x 35$/h x 2000 jue-


gos/ao = $560 x 2,000 juegos/ao = $ 1.12 millones/ao

Costos totales: ( $8.6 millones/ao + $ 1.12 millones/ao


) x 3 aos =$9.72 millones/ao x 3 aos = $29.16 millones

Beneficios: $120 millones - $29.16 millones = $90.84 millones

El monto mximo a pagar para obtener la licitacin es en-


tonces de de $90.84 millones, equivalente al beneficio esti-
mado a obtener en los 3 aos de operacin.

Entrevistador: El resultado es correcto. Quisiera


que analices si la empresa podra estar dispuesta a
pagar una cifra mayor a los $90.84 millones y cules
seran esos motivos.

Casebook Trama 2015


86

Entrevistado: La empresa puede estar dispuesta a


pagar una cifra mayor por razones estratgicas
relacionadas con las oportunidades que coment al
principio. Este proyecto sirve como entrada a la
Costa Este dando la posibilidad de una futura
expansin regional. Otro posible objetivo estratgico
es el de aumentar su Market Share tomando control
del territorio que actualmente controla un
competidor. Por otra parte hay que analizar si el
cliente tiene la capacidad de ir a prdida para
cumplir dichos objetivos estratgicos.

Evaluacin de la resolucin del caso


87

Caso N 6: SOUTH AMERICAN AIRLINES (SAA)

Introduccin

En un esfuerzo por aumentar la venta durante vuelos, Sou-


th American Airlines (SAA) est contemplando realizar una
alianza con Starcoffee, un lder regional en el mercado del
caf, para vender algunas variedades de sus productos en
los vuelos para incrementar las ventas.

Desde la perspectiva de SAA, una alianza de ese tipo sera


rentable?

Si el candidato no menciona, preguntar si habra otros obje-


tivos ms all de la rentabilidad (ej. campaa de marketing
conjunta). Sin embargo, despus de listar los otros objetivos,
focalizar el caso en la rentabilidad.
Ahora el candidato debera solicitar informacin adicional so-
bre las rutas de la aerolnea, los productos que vende actual-
mente, su clientela, sus objetivos financieros con este pro-
yecto, as como tambin informacin sobre los productos de
Starcoffe que planea vender.

Informacin sobre las rutas

75% de las rutas de SAA son de menos de dos horas. Re-


cientemente, SAA ha aadido dos rutas a su portfolio: una
que conecta Buenos Aires con San Pablo, y la otra que co-
necta Lima con Santiago de Chile. Estas rutas comprenden
el 25% restante. Cada uno de estos vuelos es directo y dura
aproximadamente seis horas.
SAA realiza 80 vuelos por da. (60 son vuelos cortos y 20 son
largos)
SAA ofrece refrescos gratuitos, agua embotellada, t, caf
y caf descafeinado en todos sus vuelos, sin importar su
duracin o el horario del vuelo. En los vuelos de madrugada,

Casebook Trama 2015


88

SAA cobra por jugos, rosquillas y desayunos. Manes, pa-


pas fritas y frutas son solamente ofrecidas en las dos rutas
entre pases. Cerveza, vino y champagne se venden en to-
dos los vuelos. Estos items se venden a un valor un 60%
superior al costo.

Informacin sobre los clientes

El candidato puede preguntar algo sobre la flota de SAA, la


clientela, la tasa de ocupacin y las ventas promedio por vuelo.

Todos los aviones de SAA son del mismo tamao, con una
capacidad para 250 pasajeros. La clientela est formada por
viajeros de negocios que realizan viajes de un da. La tasa
de ocupacin promedio es de 90%. Como mayoritariamen-
te se trata de viajeros de negocios, estos realizan compras
durante el vuelo.
El nmero promedio de ventas por vuelo depende del ho-
rario del da y de la duracin del vuelo. Las ventas por la
maana y temprano a la tarde para vuelos cortos es de 50$
por vuelo. Las ventas en los vuelos largos y tempranos pro-
median los 100$ por vuelo. Por otro lado, las ventas para
vuelos nocturnos promedian los 150$ para vuelos cortos y
200$ para los largos.
El pasajero promedio (de aquellos que compran algn
item) gasta 5$.
De los 80 vuelos diarios, 40 son de maana y tarde y 40
son de noche. En trminos de corto y largo, 75% de los
vuelos tempranos son cortos y 75% de los vuelos nocturnos
son cortos.

El candidato debera resumir la informacin en una tabla y


tambin resumir lo siguiente:

En un da dado, las ventas de SAA en vuelo suman 9000$


y con un costo del 60%, el costo diario es de 5400$ y el
margen de 3600$.
89

El entrevistador le puede pedir al candidato que obtenga in-


dicios a partir de los clculos para continuar con el caso. El
candidato debera estimar las ventas en cada tipo de vuelos
(corto temprano, largo temprano, corto tarde y largo tarde)

Posibles indicios

Con un 90% de ocupacin, se tienen 225 potenciales


clientes por vuelo. Se observar primero los vuelos cortos/
temprano. Diez bebidas se venden cada uno de estos trein-
ta vuelos. Se asume que cada bebida es comprada por un
pasajero diferente. Como son vuelos tempranos tomados
mayoritariamente por viajeros de negocios que no son muy
sensibles al precio, se supone que solamente el 10% de los
215 pasajeros restantes comprarn un producto de Starco-
ffe. Esta hiptesis se basa en una serie de factores. En pri-
mer lugar, muchos pasajeros que toman un vuelo temprano
prefieren tomar una siesta durante el vuelo en lugar de tomar
caf. En segundo lugar, para una buena cantidad de pasa-
jeros, el caf gratuito ofrecido actualmente satisface el de-
seo de consumidor de cafena. En tercer lugar, es probable
que varios pasajeros compren un producto de Starcoffee en
la terminal mientras esperan a abordar el avin. Por ende
se puede asumir que 10% 21 pasajeros son potenciales
clientes en un vuelo corto/temprano.
Los vuelos cortos/nocturnos probablemente tendrn un n-
mero menor de consumidores de productos de Starcoffee.
Se estima este nmero en un 5% de los pasajeros que no

Casebook Trama 2015


90

ordenan una bebida. Dos factores justifican este supuesto.


El primero, la venta de caf decae a medida que avanza el
da. Bebidas sustitutas como gaseosas y agua embotellada
contribuyen a esta cada, adems de que la necesidad de
mantenerse despierto tambin disminuye. Segundo, el caf
es una bebida que acompaa al desayuno de mejor manera
que el almuerzo, merienda o cena. Por lo tanto 5% de 205 da
10 potenciales clientes.
Los vuelos largos/tempranos, promedian las 30 ventas de bebi-
das por vuelos. Eso deja 195 pasajeros, asumiendo que los que
compran jugos no compraran caf. Aunque estos vuelos son
usualmente tres veces ms largos, se asume que solo el 10%
de los pasajeros restantes compraran un producto de Starco-
ffee. Las mismas razones mencionadas para los vuelos cortos/
temprano justifican el nmero. En primer lugar, el viaje permite
un espacio ms prolongado para dormir. Tambin el caf gratui-
to es un sustituto para muchos. Adicionalmente, la oportunidad
de beber mltiples vasos en forma gratuita es un atractivo extra
para los clientes. Entonces, 10% implica 19 pasajeros.
Por ltimo se analizan los vuelos largos/tarde. Con cuarenta
bebidas vendidas por vuelo, hay 185 pasajeros que podran
comprar caf. Una vez ms, se estima que un 5% de los pa-
sajeros que no ordenan una bebida comprarn caf. El caf
premium no es una bebida que se tome en horas nocturnas,
especialmente con la posibilidad de accede a bebidas que
son ms aptas para el horario. Por lo tanto, el nmero de pa-
sajeros que compraran caf es 5% de 185, que da 9. La tabla
siguiente resume lo discutido.
91

El candidato debe llegar al punto de decir cuntos cafs pue-


den ser vendidos en un da dado. En este ejemplo son 1210
cafs en un da, pero el candidato puede llegar a un nmero
diferente dependiendo de sus hiptesis.

Aqu el entrevistador le debe preguntar al candidato si la esti-


macin es razonable. Cunta ganancia puede SAA obtener
de la venta de los productos de Starcoffee en sus vuelos?

Ahora el candidato debera resumir y realizar una recomenda-


cin respecto a la alianza.

En la recomendacin, el candidato debe tener en cuenta lo


siguiente

El candidato debe preguntar qu vende la competencia en


sus vuelos
El candidato debe considerar que SAA ahorra dinero en la
bebida gratuita por cada producto de Starcoffe que vende
El candidato debe considerar si los pasajeros cambiaran a
SAA porque ahora sirve caf de Starcoffe en todos sus vuelos
El candidato debe considerar que usualmente los viajeros
de negocios no pagan de su bolsillo, sino que la compaa
en la que trabajan se hace cargo. Eso implica que no duda-
rn en elegir el caf de Starcoffee.
El candidato debe considerar zonas horarias y jet-lag. El con-
sumo de cafena es comn entre los viajeros de negocios.

Evaluacin de rendimiento

Rendimiento excelente: hacer los clculos con hiptesis razo-


nables y hacer una recomendacin considerando otros an-
lisis que pueden ser realizados
Rendimiento promedio: hacer los clculos con hiptesis razo-
nables y hacer una recomendacin solamente basada en
nmeros, sin considerar otros factores
Rendimiento pobre: dificultad para desarrollar hiptesis razona-
bles para llegar a un resultado numrico de ventas por vuelo.

Casebook Trama 2015


92

Caso N 7: BeerBusch

Introduccin

Nuestro cliente es Beer Busch. Su producto estrella es Bu-


dbeer, marca lder en Estados Unidos. Ellos quieren saber
si se debera cambiar el envase de Budbeer de botellas de
vidrio a botellas de plstico. Cules son las ventajas y des-
ventajas de dicho cambio?

El objetivo de Budbeer es el de aumentar sus ganancias y


el market share. Actualmente, beer Busch y sus treinta pro-
ductos tienen el 46 por ciento del mercado domstico. Bud
y Bud Light tienen el 25 por ciento del mercado total.

Ahora el candidato debera empezar por listar algunas ventajas


y desventajas del cambio. Si el candidato se queda demasia-
do tiempo dando hiptesis, el entrevistador debe preguntarle
si es importante estimar el tamao del mercado cervecero es-
tadounidense (idealmente, el candidato debera darse cuenta
por si mismo que el tamao del mercado es un requisito).

Posible estimacin del mercado de cerveza en EEUU

Empecemos por tomarnos un minuto para determinar el ta-


mao del mercado cervecero de Estados Unidos. Me gus-
tara descomponerlo por generacin. Asumir que hay 320
millones de estadounidenses. Tambin asumir que estn
distribuidos equitativamente por generacin. Entonces hay
80 millones de personas por generacin.

Estimar que la gente en la primera generacin, entre 0 y


20 aos, no toma cerveza.
La segunda generacin, de entre 21 y 40 aos, es proba-
blemente aquella con mayor consumo. Asumir que el 80
por ciento de ellos toma cerveza y consume 5 cervezas por
semana o 250 por ao. Eso es 64 millones de bebedores por
93

250 cervezas, lo que es igual a 16 mil millones de cervezas


por ao.
Para aquellos en la tercera generacin voy a estimar que
el 50 por ciento toma cerveza y que toman 3 cervezas por
semana o 150 por ao. Eso es igual a 6 mil millones de cer-
vezas.
En la ltima generacin, voy a asumir que el 25 por ciento
de ellos toma cerveza, siendo un total de 20 millones y que
consumen dos cervezas por semana o 2 mil millones de cer-
vezas por ao.

Estos totales incluyen las cervezas que se toman en los ho-


gares y en pblico, como bares y restaurantes. El total es de
24 mil millones de cervezas.

Ahora el candidato debera traducir el nmero de cervezas en


trminos de botellas.

Posible estimacin

Asumir que las botellas y las latas se venden en igual ma-


nera, entonces asumir una separacin 50/50 latas a bote-
llas. Eso significa que estamos hablando de 12 mil millones
de botellas de cerveza. Ahora, se mencion que Bud y Bud
Light representan el 25 por ciento del mercado, entonces
estamos viendo el 25 por ciento de 12 mil millones, lo que es
3 mil millones de Bud y Bud light vendidas cada ao.

Casebook Trama 2015


94

Ahora el candidato puede responder preguntas respecto al


material de la botella como las siguientes:

Tiene el plstico la misma vida til que el vidrio o


el aluminio?
Si, si la botella est coloreada.

Mantiene la botella plstica el fro como el vidrio o


el aluminio?Afecta el sabor?
Se mantiene fra tanto tiempo como la de vidrio y
ms fra y por ms tiempo que el aluminio. Y la
botella no afecta el sabor.

Algn competidor hizo el cambio a botellas


de plstico?
Se asume que no. Se asume que el peso del
plstico es la mitad que aquel del vidrio. De hecho,
el plstico pesa un sptimo de lo que pesa el vidrio.

Entonces es ms liviano por lo que se va a ahorrar


en gastos de transporte pero, es ms barato?
S, cuesta aproximadamente la mitad. Se asume
que para Bud cuesta 8 centavos por cada botella
de vidrio y 4 centavos por cada botella de plstico.

Tienen que redisear la planta?


Van a haber modificaciones. La cerveza esta hir
viendo cuando es embotellada. Entonces entre el
calor y la presin en que la cerveza es envasada, a
menos que se la enfre primero, la botella de
plstico podra deformarse. Adems, el sistema de
tapado es distinto. Las botellas plsticas tienen
tapas a rosca.

Cul es el costo esperado del rediseo de planta?


Quince millones de dlares.

El entrevistador le puede preguntar al candidato: Espers


que aumenten las ventas de Bud por cambiar el envase?
95

Posible respuesta

Si se cambia de vidrio a plstico completamente se per-


dera lealtad de los consumidores de cerveza de la mediana
edad, entonces se propone ofrecerlo como una alternativa
de packaging. La gente podra conseguir ambos tipos de
envase.
Hacer un estudio de mercado por dieciocho meses en ciu-
dades seccionadas.

El entrevistador podra preguntar: Cules son los aspectos


econmicos de esta decisin?

Posible respuesta

Con 3 mil millones de Bud vendidas por ao. El precio de


la botella de plstico es de cuatro centavos menos que la
de vidrio. Entonces 4 centavos por 3 mil millones equivalen
a 120 millones lo quees significativamente mayor que los 15
millones que costara el rediseo de planta. Econmicamen-
te, tiene sentido aunque sea hacer una prueba de mercado.

El entrevistador puede pedirle al candidato que resuma.

Ahora el candidato puede hacer una recomendacin y decidir


si la compaa debera cambiar el envase de Budbeer de bo-
tella de vidrio a botella de plstico

El entrevistador puede preguntarle al candidato: Cules son


las ventajas y las desventajas?

Posible respuesta

Como el objetivo es incrementar las ganancias, me gus-


tara separar las ventajas en dos campos, reduccin de
costos y aumentar los ingresos. Empecemos por los aho-
rros en los costos. Primero, la botella de plstico es ms
barata que la de vidrio. Se ahorra dinero en la produccin

Casebook Trama 2015


96

ya que los costos fijos deberan bajar. Segundo, los cos-


tos de distribucin sern menores debido al menor peso
de las botellas de plstico respecto de la de vidrio. Ade-
ms, habran menos rupturas. Se podran aumentar las
ventas porque se podra vender cerveza en botellas ms
grandes. Hasta se podra aumentar el market share con
los consumidores de edades entre veintiuno y treintainue-
ve aos.

La desventaja puede radicarse en la imagen. Pero si se


ofrece como una alternativa de packaging, entonces los
tradicionalistas no se veran afectados. La otra desven-
taja puede ser que los jvenes consumidores engullan la
cerveza como lo hacen con las gaseosas.

Evaluacin de rendimiento

Rendimiento excelente: hacer los clculos con supuestos ra-


zonables y hacer una recomendacin teniendo en cuenta
otros anlisis que se pueden hacer como se mencion
Rendimiento medio: hacer los clculos con supuestos razo-
nables y hacer una recomendacin basada slo en esos
Rendimiento pobre: dificultad para desarrollar suposiciones
para llegar a un resultado numrico de ventajas econmicas.

Caso N 8: Steel

Introduccin

Una empresa familiar que produce y vende productos de


acero, ha crecido en volumen de ventas y en facturacin
en los ltimos 3 aos, pero sus utilidades estn estanca-
das. Acaba de ser comprada por un grupo internacional que
quiere saber qu debe hacer para volver a los niveles de
rentabilidad anteriores.
97

Gua para el entrevistador

1. El entrevistado debe comenzar por la ecuacin de Pro-


fits (puede llegar por caminos alternativos como pensar
en los factores internos y externos, pero al final llegar a
la misma conclusin)

Profits = Precio x Volumen (ton) Costo Variable * Volumen (ton) Costo Fijo

Debe descartar la primer parte (precio, volumen) ya que


las ventas crecen en volumen y facturacin, y focalizarse
en costos.

2. Luego, debe analizar los costos con cierta granularidad


(entrevistador debe guiarlo si hace preguntas demasiado
genricas, e.g. si pregunta si han subido los costos varia-
bles, debe guiarlo a pensar en sus componentes). Debe-
ra llegar a una visin como la siguiente:

*Es probable que la palanca de mix no sea fcilmente identificada


por el entrevistado, por lo que el entrevistador debe ayudarlo a con-
siderarla en caso que no la identifique. Sugerencia: hacer un paralelo
con otra industria como Bienes de Consumo.

Casebook Trama 2015


98

Informacin adicional entregada dependiendo de las pre-


guntas del entrevistado

Costos fijos ~20% del total de costos


Materia prima (chatarra) y energa representan ~90% del
costo variable
No ha habido aumento de salarios, ni variaciones signi-
ficativas de la productividad, ni cambios significativos de
la estructura
El precio de la chatarra y la energa no ha variado
El rendimiento de la chatarra y la eficiencia energtica
de la planta se han mantenido razonablemente constantes

3. El entrevistado debe concluir que el problema est en


el mix de productos. Hay dos lneas de productos (barras
y mallas). El entrevistador debe pedir calcular la rentabili-
dad marginal de cada lnea con los siguientes datos:
99

4. Con esto se debe concluir que las mallas si o si han ca-


do en volumen (no solo en volumen relativo respecto a las
barras, sino en volumen absoluto, dado que si no las utili-
dades de la empresa habras crecido, mismo si se hubiese
mantenido constante). Aqu el entrevistador debe preguntar
por razones que pueden haberlo causado. El entrevistado
debera llegar a una estructura como la siguiente:

Informacin adicional a ser entregada dependiendo de las


preguntas del entrevistado (y notas)

Empresas que actan en mallas tambin lo hacen en


barras (sirve para descartar la entrada de un nuevo com-
petidor como causa ya que si no hubiesen cado las ba-
rras tambin)
Actuales competidores han hecho movimientos para
capturar share
El cliente no ha sufrido por calidad de sus productos ni
ha cambiado su poltica de precios
La capacidad instalada de mallas no ha cambiado
La causa de la falta de chatarra debe ser descartada
dado que en ese caso no se podran fabricar tampoco barras
Mercado (entrevistado debe calcular el crecimiento,
para ver si mallas han cado). El entrevistado debe ver
que las mallas crecen ms y entonces descartar la causa

Casebook Trama 2015


100

del menor crecimiento de mercado:

o Mercado total (mallas + barras): 900k ton en el


Ao 1 y crece al 3% anual
o Mercado de barras: 750k ton en Ao 1
o Mercado de mallas: 165k tonen Ao 3

Nota: calculados en negrita; importante considerar 2 aos (del 1 al 2


y del 2 al 3); se puede considerar los % como no acumulativos para
simplificar ya que son porcentajes chicos

5. El entrevistado debe concluir que han decidido vender


menos mallas por una restriccin de energa. El entrevis-
tador debe preguntar si ha sido una decisin lgica. Para
eso el entrevistado debe calcular la rentabilidad marginal
por unidad de energa consumida por cada producto
101

El entrevistador debe concluir que la decisin fue co-


rrecta y que si hay una restriccin de energa lo mejor es
producir ms barras, no pudiendo volver a los niveles de
rentabilidad originales.

6. Por ltimo, el entrevistador debe preguntar qu solu-


ciones pueden existir y los prximos pasos asociados a
cada una. El entrevistado debera identificar algunas de
las siguiente:

Evaluacin de performance

Si el entrevistado no ha llegado al paso 4, est por debajo


del nivel mnimo del caso. Como mnimo debera identificar
las causas (o la gran mayora) y descartar las que no aplican
para focalizar en la energa, con una gua moderada (espe-
cialmente para identificar el mix de producto como causa).

En caso de identificar que en caso de una restriccin la de-


cisin fue correcta, el entrevistado est por encima de la
media. Por ltimo, en caso de pensar en potenciales solu-
ciones y sus prximos pasos tendr un desempeo sobre-
saliente.
No es un ejercicio pesado desde los nmeros, pero puede

Casebook Trama 2015


102

trabarse si no se redondean / simplifican los clculos consu-


miendo mucho tiempo.

Caso N 9: Salud

Introduccin

Un grupo de inversores ha comprado una empresa en


Brasil que tiene 4 unidades de negocio

Salud: planes de salud para empresas y personas (similar


a una obra social)
Asistencia: administracin de servicios de asistencia para
aseguradoras (e.g. gra remolque para seguro de auto,
servicio de electricista o plomero para seguro hogar, etc.)
Odonto: planes odontolgicos, similares a los de salud
pero con cobertura de odontologa
Homecare: servicios de atencin e internacin domiciliaria

El grupo design un nuevo CEO y nos ha contratado para


definir un plan agresivo de crecimiento. El CEO agend una
reunin con todo el directorio para discutir las hiptesis ma-
ana y tenemos tiempo de profundizar en una UN sola. Para
eso necesitamos definir rpido en cul focalizar y armar la
hiptesis de crecimiento.

Gua para el entrevistador

1. Para definir en qu UN focalizar, el entrevistado debe pe-


dir Info de tamao y rentabilidad

Nota: los datos de Utilidades (en rojo) deben ser calculados por el entrevistado
103

En conclusin, hay que focalizar en Salud por tamao y uti-


lidades generadas.

2. El primer paso para entender las oportunidades de creci-


miento, primer debe entenderse el mercado, el posiciona-
miento de la empresa y el entorno competitivo. Inicialmen-
te, el entrevistado debe estimar el tamao del mercado de
planes de salud. Para hacerlo es importante separar en
niveles socio-econmicos. El entrevistador le entrega los
siguientes datos para calcular el mercado anual en R$M.

Nota 1: Poblacin total de 190M de habitantes (puede redondear a


200M); Nota 2: en rojo los valores que deben calcularse; Nota 3: si
se hacen los clculos uno a uno puede consumir mucho tiempo y
llevar a errores, por lo que aqu se muestran sacando los factores
comunes para simplificar

Con estos datos el entrevistado puede ver que el market


share de la empresa es de ~0.8%, pero no necesariamen-
te que tiene un posicionamiento muy alejado del lder ya
que no se sabe que tan fragmentado est le mercado. Si el
entrevistado pregunta por el resto de los competidores, el
entrevistador le entrega la siguiente informacin.

El Player A tiene una facturacin anual de R$5000M


El Player B es el 80% del tamao del Player A

Casebook Trama 2015


104

El Player C es el 50% del tamao del Player A


Nuestro cliente es el 4 player en tamao
Luego, todo el resto de los players estn muy fragmentados

3. El entrevistado debe concluir que dada la fragmentacin


del mercado (4 mayores players tienen ~20% del merca-
do), debe haber espacio para identificar nichos (segmen-
tos, regiones, etc) donde el cliente pueda focalizarse.
Si el entrevistado pregunta por la segmentacin del mer-
cado, el entrevistador debe indicar que existen dos seg-
mentos bien marcados en donde actan las empresas en
mayor o menor medida

El posicionamiento de los principales players en cada seg-


mento es as (dar en caso que el entrevistado pregunte)
105

Si el entrevistado pregunta por la distribucin regional, el


entrevistador debe indicar que el mercado puede dividirse
en 5 regiones y dar la siguiente informacin conforme el
entrevistado la pregunte

El entrevistado debera inferir que los mercados Sur y


Nordeste son aquellos con mayor potencial de crecimiento
para el cliente. A continuacin podra comenzar a elaborar
en las principales iniciativas de crecimiento que encuentra,
destacando:

Casebook Trama 2015


106

Evaluacin de performance

El paso 1 determina la capacidad de priorizacin del en-


trevistado. Es indispensable que el entrevistado busque
una priorizacin inicial considerando el valor relativo para
la empresa de cada una de las UN. Debera identificar las
utilidades como factor de decisin.

En el paso 2, el entrevistado debera por lo menos mostrar


recursos para estimar el mercado desde valores generales,
y proponer una clara distincin de distintas categoras de
clientes (niveles socio-econmicos). Estar por debajo de
la media si no logra buscar un camino de resolucin mate-
mtico simplificado (ie. Factor comn, redondeo/aproxima-
cin).
107

Avanzando en el paso 3, el entrevistado debera entender


la importancia de entender subsegmentos de mercado
(prepago y pospago) y distribucin regional. Un entrevis-
tado en la media debera poder interpretar el potencial de
cada uno de ellos.

Un buen desarrollo del caso implicara que el entrevistado


llegue a priorizar los productos y mercados de crecimiento,
y proponer aunque sea una opcin preferente de creci-
miento y eventualmente poder comentar en otra iniciativa
alternativa. Debera poder explicar los racionales para esa
iniciativa, y poder comentar en los prximos pasos o ma-
neras de desarrollar la iniciativa, entendiendo los riesgos o
dificultad de ello.

Casebook Trama 2015

You might also like